Chest Wall / Abdominal Wall / Gynecomastia Flashcards

1
Q

An 18-year-old woman is evaluated for chest wall and breast asymmetry. She has a history of pectus excavatum and underwent surgery as a child for placement of a correction bar, which was subsequently removed 3 years after insertion. Since the removal of the bar, she has noticed progressive recurrence of her chest wall deformity with associated breast asymmetry. Physical examination shows a thin patient with a 4-cm deep concavity involving the lower end of the sternum and medial distortion of the right breast. The patient denies shortness of breath or chest pain. Which of the following treatment options is the most appropriate recommendation for contour improvement?

A) Autologous transfer of fat tissue to the sternal defect and right breast
B) Implantation of a customized silicone elastomer device with concurrent augmentation mammaplasty
C) Injection transplantation of cultured autologous chondrocytes
D) Placement of bilateral silicone breast implants greater than 550 cc
E) Reinsertion of the correction bar with sternal wiring

A

The correct response is Option B.

When considering how best to treat pectus excavatum in the female patient, it is important to recognize that the majority of the contour deformity is due to the thoracic concavity with only a small portion of the deformity due to actual breast tissue hypoplasia. However, reinsertion of a correction bar is not always successful in correcting the thoracic deformity in post-adolescent patients whose bones have ossified, and is only done in extreme cases. Augmentation mammaplasty is also unsuccessful, regardless of implant volume, since, as stated above, only a small portion of the defect is actually caused by breast tissue asymmetry. Although some might consider autologous fat grafting a viable option for correcting both the chest wall defect and the hypoplastic breast, there have been reports that indicate that injecting between the tight adhesions of presternal skin and bone is technically challenging and rarely successful. Injection of cultured autologous chondrocytes, which is still in the early stages of research and development, would prove to be equally technically challenging, and at this time, de novo generation of cartilage or fat for injection into soft-tissue defects is still unproven. As a result, customized silicone elastomer implants are commonly used in conjunction with augmentation mammaplasty to provide consistent and reliable correction of pectus excavatum and breast asymmetry in the female patient. Studies have shown that placing a custom sternal prosthesis and bilateral breast implants during one surgery is both safe and effective in producing an improved aesthetic result.

2018

How well did you know this?
1
Not at all
2
3
4
5
Perfectly
2
Q

A 5-year-old boy is evaluated for a cerebrospinal fluid leak and a 3 × 3-cm area of wound dehiscence involving the posterior trunk following tethered cord repair. Which of the following is the most appropriate method for reconstructing the wound?

A) Gluteal muscle flap and skin advancement flap
B) Latissimus muscle flap and skin advancement flap
C) Local fascial flap and skin advancement flap
D) Skin advancement flap only
E) Split-thickness skin graft

A

The correct response is Option C.

The most appropriate method to reconstruct the wound is a local fascial flap and skin advancement flap. The major principle of tethered cord and myelomeningocele repair is to obtain a well-vascularized layer of soft-tissue coverage between the dural and skin closures. The fascia overlying the paraspinous muscles can be turned over as flaps to cover the underlying dural repair. This vascularized soft-tissue layer will minimize the risk of cerebrospinal fluid leak by reinforcing the dural repair. In addition, the fascial flaps will prevent contact with cutaneous bacteria and subsequent meningitis if either the dural repair or skin repair breaks down.

A split-thickness skin graft over the dura would not adequately protect the spinal cord. Closing the skin directly over the dural repair using skin advancement flaps would place the child at risk for meningitis in the event of a cerebrospinal fluid leak or if wound breakdown occurred along the incision line of the widely undermined skin flaps.

The use of a regional gluteal or latissimus muscle flap to cover the dural repair is unnecessary because local tissue (paraspinous muscle fascia) is available. Harvesting the gluteal or latissimus muscles also may cause significant donor site morbidity in a child already at risk for ambulatory problems from a neurological deficit.

2018

How well did you know this?
1
Not at all
2
3
4
5
Perfectly
3
Q

A 30-year-old woman is evaluated for a mass on the anterior abdominal wall that has been growing over the past several months. Imaging and examination of the specimen obtained on biopsy confirm a diagnosis of desmoid-type fibromatosis arising from the anterior abdominal musculature. Which of the following is the most accurate statement regarding this condition?

A) It is associated with previous radiation exposure
B) It is locally aggressive with remote risk of metastatic disease
C) It usually arises from an untreated lipoma
D) Primary treatment is combination chemotherapy/radiation
E) There is an autosomal recessive genetic inheritance pattern

A

The correct response is Option B.

This patient has a desmoid tumor, a rare, locally infiltrative mesenchymal neoplasm that is most commonly found in adolescents and young adults. It does not metastasize but can have a very unpredictable and aggressive natural history. Treatment can range from close observation (as some tumors will self-involute) to radical excision with wide margins. Traditionally, surgical excision can be difficult because of the high risk of local recurrence.

Desmoid tumors are not associated with chemical or radiation exposures, or lipomas.

Metastatic disease is not a characteristic of desmoid tumors.

Although treatment modalities for desmoids are evolving, they are not treated with chemotherapy/radiation therapy as a primary modality. Primary treatment is frequently observation for small or slowly changing tumors. For rapidly growing tumors, radical excision is frequently performed.

There does not appear to be a genetically inherited component for desmoid tumors, although they are associated with random mutations of the ?-catenin (CTNNB1) and adenomatous polyposis coli (APC) genes.

2018

How well did you know this?
1
Not at all
2
3
4
5
Perfectly
4
Q

An 18-year-old woman comes to the office to discuss her congenital breast asymmetry. Physical examination shows asymmetry in the size and shape of the right breast and nipple-areola complex compared with the left breast. Additionally, there is an absence of the right anterior axillary fold. A photograph is shown. Which of the following is the most likely origin of the absent muscle in this patient?

A) Ectoderm
B) Endoderm
C) Mesoderm
D) Neuroectoderm
E) Notochord

A

The correct response is Option C.

The patient above has Poland syndrome. Poland syndrome represents a spectrum of congenital chest wall anomalies ranging from the simple form (depicted in the preoperative photograph) to complex. The pathognomonic feature is the absence of the sternocostal head of the pectoralis major muscle. A variety of other ipsilateral chest wall and upper extremity malformations may be present, including absence of ribs two through five, foreshortening of the limb, brachysyndactyly, hypoplasia/aplasia of the breast and nipple-areola complex, and absence/hypotrophy of various trunk muscles including the latissimus dorsi, serratus anterior and external oblique. Although a few familial cases have been reported, Poland syndrome is believed to be sporadic in nature with an incidence of 1:100,000. There is a male-to-female predilection of 3:1, with a right-sided predominance in boys of 2:1. The patient is shown two months after reconstruction with a right latissimus flap and bilateral silicone gel-filled implants.

The trunk develops from the fusion of the ectoderm/neural crest layer with the somatic mesoderm during the fourth week of gestation. Mesoderm gives rise to somites, which differentiate into the dorsolateral dermomyotome and the ventromedial sclerotome. The dermomyotome gives rise to the musculature of the trunk and extremities. The sclerotome surrounds the developing spinal cord and notochord and forms the vertebral bodies and spinal skeletal structure. The notochord acts as a pathway for the development of the spinal cord and vertebral bodies. The notochord degenerates as the vertebral bodies develop and becomes the nucleus pulposus of the intervertebral disks.

The exact pathophysiology of Poland syndrome has not been elucidated. One theory suggests a mechanical disruption of the embryonic blood supply in the subclavian/vertebral systems during the sixth to seventh week of gestation. Another theory suggests the etiology to be an injury or developmental failure of the mesodermal plate during the third to fourth week of gestation.

Endoderm is the primitive germ layer that gives rise to the epithelium of the respiratory and gastrointestinal tracts. Ectoderm is divided into surface ectoderm and neuroectoderm. Surface ectoderm gives rise to the epidermis, adnexal structures of the skin, and the mammary glands. Neuroectoderm becomes the central nervous system, various neural ganglia, and the branchial arches.

How well did you know this?
1
Not at all
2
3
4
5
Perfectly
5
Q

During a ventral hernia repair and abdominal wall reconstruction, a surgeon wishes to achieve full primary fascial closure over an intraperitoneal mesh. However, the fascia is under tension and requires a component separation to achieve full closure with minimal tension. Component separation technique involves which of the following?

A) Incision of the anterior rectus sheath
B) Internal oblique aponeurotomy
C) Ligation of periumbilical perforators
D) Plication of the linea semilunaris
E) Preservation of intercostal nerves

A

The correct response is Option E.

Preservation of segmental intercostal nerves is a critical aspect of component separation.

Component separation is a powerful technique used during abdominal wall reconstruction to advance the abdominal fascia towards the midline and allow for primary closure with reduced tension.

A component separation is performed by making an incision longitudinally in the external oblique aponeurosis, just lateral to the linea semilunaris. Only the fascia is cut, not the external oblique muscle. This allows for advancement of a myofascial complex consisting of the rectus abdominus muscle, internal oblique muscle and fascia, and transversalis muscle and fascia.

The segmental intercostal nerves that supply the anterior abdominal wall run between the internal oblique and the transversalis muscle. They are deeper than the incision and dissection plane with component separation, and are thus protected from injury. This maintains abdominal wall tone in patients undergoing this procedure.

External oblique aponeurotomy, not internal oblique aponeurotomy, is involved in component separation. Incision of the anterior rectus sheath is not part of component separation, although some modifications include incision of the posterior rectus sheath to release more fascia available for primary closure. Plication of the linea semilunaris is not a part of component separation. Ligation of periumbilical perforators is commonly performed in the standard, open approach to component separation, but it is not considered a critical component of the procedure, as it is possible to perform a perforator-sparing component separation.

2018

How well did you know this?
1
Not at all
2
3
4
5
Perfectly
6
Q

An 80-year-old man comes to the office because of recurrent squamous cell carcinoma of the cervical skin. Following reconstruction with a pectoralis myocutaneous flap, the distal half of the skin paddle appears ischemic and eventually exhibits necrosis and full-thickness skin loss. Transection of which of the following structures is most likely responsible for this complication?

A) Lateral thoracic artery
B) Medial pectoral vein
C) Posterior intercostal vasculature
D) Thoracodorsal artery
E) Transverse cervical vessel

A

The correct response is Option A.

The pectoralis major muscle has three major blood supplies. In general, the internal mammary perforators and the thoracoacromial vessels are dominant, with additional perfusion through the lateral thoracic artery. Turnover pectoralis flaps for sternal reconstruction based on the internal mammary perforators are generally well perfused. Likewise, in most situations a pedicled myofascial or myocutaneous flap based on the thoracoacromial vessels (with the other blood supplies ligated) is robust and can be used for most types of head and neck reconstruction. Although described earlier, the flap became the “workhorse” of head and neck reconstruction after Ariyan’s classic article in 1979. Versus a free tissue transfer, the pectoralis flap has many detractors, including being tethered to its pedicle. In about 6% of cases, the lateral thoracic vessel is the dominant pedicle to the flap and needs to be incorporated for maximal perfusion for head and neck reconstruction. This of course further decreases some of the mobility of the flap. This phenomenon is usually easily discernible by the larger caliber of the lateral thoracic vessels versus the thoracoacromial vessels. In this case, this was the most likely injured vessel, causing the described outcome.

The transverse cervical vessels do supply the posterior thorax, and the intercostal vessels do supply the anterior chest, including breast tissue, but neither gives important perfusion to the pectoralis flap. The lateral pectoral nerve travels with the thoracoacromial vessels and, if not checked when turning the flap, can kink the vascular pedicle and, if not severed, does not allow the flap to decrease in size because of lack of denervation of the muscle. The medial pectoral nerve is another innervation to the pectoralis major muscle and is usually severed during flap elevation. The thoracodorsal artery does not supply the pectoralis muscle.

2018

How well did you know this?
1
Not at all
2
3
4
5
Perfectly
7
Q

A 59-year-old man is scheduled for reconstruction of a central abdominal wall hernia measuring 10 cm wide x 30 cm long. A surgical approach using posterior component separation is planned. Which of the following locations for fascial incision most accurately describes the technical considerations of this procedure?

A) Across the lateral intercostal neurovascular bundles
B) Along the mid-axillary line
C) Medial to the linea semilunaris
D) Parallel to the subcostal border
E) Vertical bisection of the rectus muscles

A

The correct response is Option C.

Component separation of the abdominal wall was initially described for the anterior components, that is, those which are located anterior to the rectus muscle. A more recent development involves component separation of the layers located posterior to the rectus fascia. The technique of posterior component separation begins with a vertical incision of the posterior rectus sheath 0.5 cm medial to the linea semilunaris and continues laterally in the avascular plane posterior to the transversalis muscle. It can extend as far posteriorly as the psoas muscle if needed. In the event that the posterior layer cannot be approximated in the midline, an interposition patch of omental fat, hernia sac, or absorbable mesh is used. Concurrent use of non-absorbable mesh to reconstruct deficient anterior layers may be used in conjunction with the posterior separation technique as long as the mesh is separated from viscera with an intact posterior layer. A benefit of the posterior separation technique is the preservation of the lateral neurovascular bundles preserving the dynamic function of the rectus muscles. The technique may be utilized even when fascial defects are not situated in the midline or are located adjacent to bony landmarks.

2018

How well did you know this?
1
Not at all
2
3
4
5
Perfectly
8
Q

A 43-year-old woman, gravida 3, para 3, undergoes a combined panniculectomy and hernia repair. The planned hernia repair is a retrorectus repair without components separation with polypropylene mesh. Which of the following best describes an advantage of using polypropylene mesh compared with a biologic scaffold?

A) Decreased need for components separation
B) Decreased recurrence rate
C) Increased collagen cross-linking
D) Increased resistance to infection

A

The correct response is Option B.

In general, the indication to use biologic scaffolds in hernia repairs and abdominal reconstruction is in contaminated beds. Biologic scaffolds provide an intact extracellular matrix and support tissue regeneration, and are more resistant to infection than synthetic mesh, likely because of this tissue ingrowth. Biologic scaffolds, unlike synthetic mesh, are degraded over time by collagenase. This accounts for at least some of the higher recurrence rates seen with biologics compared to synthetic mesh. Cross-linking of the scaffold may provide resistance to collagenase and improve long-term stability.

Thus, better resistance to infection is a property of biologic scaffolds, as is collagen cross-linking, although the degree of cross-linking varies based on the particular scaffold. They are associated with a higher cost and an increased recurrence rate. Their high cost compared with synthetic mesh does not justify their routine use, and the recommendation for their use by the Ventral Hernia Working Group is in contaminated fields, with infected mesh and septic dehiscence.

The need for components separation is based on the properties of the hernia itself and is independent of the type of mesh used. Components separation without mesh is, however, associated with a high recurrence rate.

2018

How well did you know this?
1
Not at all
2
3
4
5
Perfectly
9
Q

In addition to the typical chest wall deformities, which of the following is the most common associated clinical presentation of patients suffering from Poland syndrome?

A) Dextrocardia
B) Lung hernia
C) Renal malformation
D) Thyroid malignancies
E) Upper extremity anomalies

A

The correct response is Option E.

Poland syndrome is the rare congenital condition describing the absence of the breast or nipple, hypoplasia of subcutaneous tissue, absence of the costosternal portion of the pectoralis major muscle, absence of the pectoralis minor muscle, and absence of costal cartilages or ribs 2, 3, and 4 or 3, 4, and 5, occasionally even rib 6. Its aesthetic and functional impairment varies between individuals and highly depends on the severity of the disease. Boys are three times as likely to be affected as girls. Incidence ranges between 1:100,000 to 1:7,000 cases per live births.

Various concomitant anomalies have been published in association with Poland syndrome that typically necessitate a multi-disciplinary approach towards the diagnosis and treatment of this congenital illness. Upper extremity anomalies, renal malformation, lung hernia, and dextrocardia have all been described in literature as related to Poland syndrome. Anomalies of the upper extremity are the most common concomitant malformation besides the chest wall defects.

2018

How well did you know this?
1
Not at all
2
3
4
5
Perfectly
10
Q

A 22-year-old man comes to the office reporting breast enlargement that began 7 years ago. Physical examination shows bilateral breast enlargement. Testicular examination is normal. BMI is 27 kg/m2. Which of the following evaluations is best for distinguishing gynecomastia from pseudogynecomastia in this patient?

A) History of medication use
B) Laboratory testing
C) Mammography
D) MRI
E) Physical examination

A

The correct response is Option E.

Gynecomastia is benign enlargement of the glandular breast tissue in a man. It is often related to factors that either results in an increase in estrogen production, or a decrease in androgen production. This can be due to the transient imbalances during puberty, or secondary to medication use, drug use (such as marijuana), testicular pathology, thyroid disease, liver disease, breast cancer, etc.

This is in contrast to pseudogynecomastia, which is enlargement of the breast due to fat deposition in the absence of glandular hypertrophy. Pseudogynecomastia is typically seen in the patient who is overweight or obese.

Pseudogynecomastia is distinguished from gynecomastia by physical examination. In true gynecomastia, one will palpate enlarged firm glandular breast tissue, as opposed to in pseudogynecomastia, where palpation will reveal a soft fatty breast throughout with no enlargement of the subareolar tissue.

History may suggest a pathologic etiology that will be confirmed on laboratory testing. Medication and drug use questioning will show if the gynecomastia is pharmacologic in origin. Mammography and MRI will aid in determining if there is a malignant etiology causing the breast tissue enlargement. However, it is not standard for gynecomastia evaluation.

2018

How well did you know this?
1
Not at all
2
3
4
5
Perfectly
11
Q

Which of the following surgical options is most important for gynecomastia patients with significant glandular hypertrophy?

A) Areolar reduction
B) Excess skin removal
C) Lateral lipectomy
D) Subtotal glandular resection
E) Ultrasonic liposuction

A

The correct response is Option D.

In patients with a significant glandular component, it is important to do a subtotal glandular resection to prevent recurrence and provide the best chance at adequate contour. Ultrasonic liposuction has been used to attempt subtotal removal of gland tissue, but it does not reliably produce a subtotal resection. Areolar reduction is usually not necessary due to the contractile nature of the tissue. Skin resection may be necessary, most commonly in the massive weight loss patient, but does not necessarily play a role in the glandular component of the resection. Lateral suction lipectomy does not correct glandular hypertrophy.

2018

How well did you know this?
1
Not at all
2
3
4
5
Perfectly
12
Q

A 48-year-old man undergoes revision ventral hernia repair. Medical history includes poorly controlled type 1 diabetes mellitus and liver transplantation 3 years ago. The duration of the operation is 6 hours and the procedure is complicated by extensive lysis of adhesions. Three days postoperatively, the patient is noted to have decreased urine output and hypotension despite crystalloid and colloid fluid resuscitation. Intrinsic renal failure is suspected. On analysis of serum and urinary electrolytes, which of the following values is expected to be normal with intrinsic renal failure in this patient?

A) BUN:serum creatinine ratio
B) Fractional excretion of sodium (FENa)
C) Urinalysis
D) Urinary sodium concentration
E) Urine osmolality

A

The correct response is Option A.

Assessment of renal failure begins with a thorough history and physical examination as well as evaluation of key laboratory measurements including complete blood count, metabolic panel, coagulation profile, urinalysis, and urine electrolytes. Early signs of renal failure may include orthostatic hypotension, tachycardia, and dry mucous membranes on examination.

Renal failure may be divided into three categories: prerenal (causes include hypovolemia, cardiac failure, sepsis), renal/intrinsic (causes include disorders of the renal parenchyma, glomerular disease, acute tubular necrosis), and postrenal (causes included renal vein occlusion, urinary tract obstruction).

Assessment of serum and urinary electrolytes and a urinalysis aid with the identification of the cause of renal failure. In prerenal failure, the urinalysis is normal, the ratio of blood urea nitrogen (BUN) to serum creatinine is elevated above 20:1, the urinary sodium concentration is less than 20 mEq/L, urine osmolality is higher than 500 mOsmol/kg H2O, and the fractional excretion of sodium (FENa), which compares the differences of the sodium and creatinine in the plasma and urine, is less than 1%.

Intrinsic renal failure usually presents with a normal BUN and serum creatinine, urinary sodium concentration higher than 40 mEq/L, urine osmolality less than 350 mOsmol/kg H2O, and an abnormal urinalysis.

Postrenal failure typically has an elevated BUN to serum creatinine, a FENa greater than 1%, and a normal urinalysis.

2018

How well did you know this?
1
Not at all
2
3
4
5
Perfectly
13
Q

A 2-year-old boy with a history of omphalocele presents for correction of a 5-cm abdominal bulge with a 3-cm widened scar over the bulge. Which of the following is the most appropriate next step in management of the bulge?

A) Anterior component separation
B) Interposition acellular dermal matrix placement
C) Interposition prosthetic mesh placement
D) Tensor fascia lata flap
E) Tissue expander placement

A

The correct response is Option A.

For most patients with omphaloceles less than 5 cm in diameter, a single operation involving a traditional anterior component separation from costal margin to iliac crest is sufficient to reduce the omphalocele and reapproximate the rectus diastasis. Extended component separations are typically only needed when the omphalocele is large and accompanied by a diaphragmatic hernia. Once the abdominal wall defect extends past 5 cm in diameter, a staged procedure involving the placement of tissue expanders and subsequent flap advancement must be considered. Autologous tissue options, such as the tensor fascia lata flap, may be needed for larger defects that can not be managed with component separation. While prosthetic or biologic mesh placement is used to correct large abdominal wall defects, it is typically used only after autologous options have failed or are not available.

2017

How well did you know this?
1
Not at all
2
3
4
5
Perfectly
14
Q

The plastic surgeon is called to the neonatal unit to evaluate a newborn with a myelomeningocele. There is an intact 4 × 4-cm sac at the lumbosacral area with minimal fluid oozing from the area. Early operative repair is indicated for which of the following reasons?

A) To decrease the need for cerebrospinal fluid shunt placement
B) To decrease the need for folic acid supplementation
C) To improve motor function return
D) To prevent bacterial meningitis
E) To prevent cerebrospinal fluid leakage

A

The correct response is Option D.

The major indication to repair a myelomeningocele defect in the early postnatal period is to prevent infection and bacterial meningitis. Great care is taken to keep the sac sterile and hydrated. Further cardiac, orthopedic, and urologic workup is often necessary, as well as evaluation for hydrocephalus. Although improvement in the return of motor function with early repair has not been shown, neurogenic bladder prognosis is improved. Leakage of cerebrospinal fluid is commonly observed regardless of timing of closure. Although folic acid supplementation has been shown to decrease the neural tube defects and myelomeningocele formation, postnatal supplementation has not been shown to be effective.

Approaches to surgical therapy for the treatment of these defects have seen a shift toward prenatal, fetal reconstructive surgery. A recent study by Adzick et al. in the New England Journal of Medicine examined the use of prenatal surgery versus postnatal surgery. In this randomized trial, the authors found a decreased need for cerebrospinal fluid shunt placement and improved motor function outcomes.

2017

How well did you know this?
1
Not at all
2
3
4
5
Perfectly
15
Q

A 10-year-old boy is brought to the office because of the findings shown in the photograph. Which of the following is the most likely diagnosis?

A) Amniotic band syndrome
B) Axillary web syndrome
C) Poland syndrome
D) Popliteal pterygium syndrome
E) Waardenburg syndrome

A

The correct response is Option C.

The patient has Poland syndrome with an unusual axillary web. The defining clinical feature is underdevelopment or absence of the sternal head of the pectoralis major muscles (seen in the photograph), but the deformity can lead to other anomalies such as absence of the nipple, the areola, or portions of the anterior chest wall. Cardiac anomalies may also be present. The ipsilateral extremity and hand are often smaller or hypoplastic, and this finding can be subtle or pronounced. The fingers are typically shorter and smaller than the contralateral side, and there may be webbing between the fingers. In severe forms, the central fingers are mere vestiges.

The other choices do not fit with this clinical photograph. Amniotic band syndrome usually leads to amputation or severe constriction of the hand or digits; this is not seen here. Waardenburg syndrome is a genetic condition that causes hearing loss and pigmentation changes to the hair, skin, and eyes. Popliteal pterygium syndrome is a condition associated with cleft lip/palate, webs of the popliteal space, and syndactyly of the toes or fingers. The axillary web seen here is not a described feature. Axillary web syndrome, or cording, is an acquired web that usually follows axillary node dissection in the treatment of breast cancer. This does not fit the clinical scenario.

2017

How well did you know this?
1
Not at all
2
3
4
5
Perfectly
16
Q

An otherwise healthy 14-year-old boy is evaluated because of bilateral breast enlargement over the past 3 months. He is worried that his friends will notice. He reports no illicit drug use and takes no medications. Physical examination shows normal hair distribution for the patient’s age, no testicular masses, and firm discs of tissue under the areola of each breast. Which of the following is the most appropriate next step in treatment?

A) Direct excision in the subareolar area, leaving a small button of tissue, and marginal breast liposuction
B) Liposuction of entire breast with use of cutting cannula under the areola
C) Reassessment in 9 months
D) Testosterone replacement
E) Wise pattern skin excision with nipple grafting

A

The correct response is Option C.

The patient appears to have benign pubertal gynecomastia, and many of these cases resolve spontaneously. The recommended treatment is observation and reassessment. Early surgery in these circumstances is usually reserved for patients experiencing more extreme psychological impact. Testosterone replacement would not be useful since by examination the patient appears to have an otherwise normal physiology. The three surgical interventions are all potential treatments for gynecomastia, but surgery is usually not recommended within the first six months of onset of gynecomastia in a pubertal male. The skin excision and nipple grafting option would be most useful if the patient had large quantities of fat and skin. The two techniques employing liposuction both might be adequate surgical techniques to address the type of gynecomastia this patient demonstrates, should surgery eventually become recommended.

2017

How well did you know this?
1
Not at all
2
3
4
5
Perfectly
17
Q

A 50-year-old man with a BMI of 36 kg/m2 comes to the office for consultation regarding gynecomastia. He takes no medications. Physical examination shows no other abnormalities. Which of the following is the most likely cause of this patient’s gynecomastia?

A) Decreased circulating estrogen
B) Decreased estrogen receptors
C) Excessive androgen receptors
D) Excessive aromatization of androgen to estrogen
E) Increased circulating androgen

A

The correct response is Option D.

Gynecomastia is benign proliferation of breast tissue in men. It is present in 40 to 50% of men over 40 years of age. It can manifest in pubertal boys and in men of advanced years. The etiology can be variable and may be due to excess circulating estrogen, decreased circulating androgens, or a deficiency of androgen receptors. However, in middle-aged and older men, it is most commonly due to the excessive aromatization of androgens to estrogens. Initial evaluation requires a detailed history and physical examination to differentiate between fatty tissue, parenchymal enlargement, and a tumor. Mammography may be useful and biopsy may be indicated in some cases.

2017

How well did you know this?
1
Not at all
2
3
4
5
Perfectly
18
Q

A 65-year-old man is prescribed leuprolide acetate for prostate cancer. Which of the following is the most likely effect the drug will have on this patient’s breasts?

A) Darkening of the nipple-areola complex
B) Decrease in size
C) Galactorrhea
D) Mastodynia
E) Petechiae

A

The correct response is Option D.

Leuprolide acetate (Lupron) is used in the treatment of certain cancers, including prostate, endometriosis, and precocious puberty. Because it is a synthetic gonadotropin-releasing hormone (GnRH), it acts as an agonist of pituitary GnRH receptors. Its ultimate effect is a lowering of estradiol and testosterone levels through downregulation of luteinizing hormone and follicle-stimulating hormone secretion.

Gynecomastia and breast tenderness are known side effects of leuprolide treatment for prostate cancer. Selective estrogen receptor modulators (SERMs), such as tamoxifen, are recommended for the prevention of gynecomastia as a result of antiandrogen monotherapy. Radiation as well has been shown to decrease the breast side effects of leuprolide acetate. Thus, an increase in size, rather than a decrease in size is expected. Darkening of the nipple areolar complex is often associated with pregnancy, but not leuprolide acetate. Galactorrhea is associated with drugs—commonly drugs that contain dopamine depleting agents, such as methyldopa, that cause receptor bock (such as Reglan), and that inhibit release, such as codeine and morphine. Histamine receptor blockade, from drugs such as cimetidine, can also cause galactorrhea. Pituitary tumors are also a cause of galactorrhea.

A thorough assessment of all medications and medical history is necessary for evaluation of a patient with gynecomastia and mastodynia.

2017

How well did you know this?
1
Not at all
2
3
4
5
Perfectly
19
Q

In bilateral component separation for abdominal wall reconstruction, which of the following points is most likely to be the area of greatest advancement?

A) Arcuate line
B) Ligament of Treitz
C) Suprapubic
D) Umbilicus
E) Xyphoid

A

The correct response is Option D.

Component separation for abdominal wall reconstruction involves release of the fascia lateral to the rectus abdominus muscles, just lateral to the semilunar line, dissecting the external oblique off the internal oblique muscles. This creates innervated musculofascial flaps that can be advanced medially for closure of ventral hernias. If the posterior rectus sheath is also dissected free, further advancements can be gained. Per rectus muscle, approximately 4 cm can be gained at the epigastric and suprapubic areas; 10 cm can be advanced at the waist. Therefore, the most advancement can be gained at the umbilicus, which is in the area of the waist. The xyphoid and ligament of Treitz are in the epigastric area, while the arcuate line is in the suprapubic area.

2017

How well did you know this?
1
Not at all
2
3
4
5
Perfectly
20
Q

A 25-year-old man returns to the operating room for closure of the abdomen 2 weeks after undergoing small-bowel resection. The abdomen was left open after the resection and treated with abdominal negative pressure wound therapy. The bowel wall edema has improved, and the fascia and rectus muscles are 30 cm apart at the level of the umbilicus. There is no evidence of contamination. Abdominal wound closure is planned. There is adequate skin for primary closure. Which of the following methods is most appropriate for fascial closure?

A) Acellular dermal matrix and negative pressure wound therapy
B) Component separation and acellular dermal matrix interposition
C) Skin grafting and negative pressure wound therapy
D) Subcutaneous tissue expansion, staged closure of skin flaps
E) Submuscular tissue expansion, staged closure of fascial flaps

A

The correct response is Option B.

Abdominal wall reconstruction after severe trauma involves evaluation of the skin and fascia. First, the surgeon needs to determine if there is sufficient skin and subcutaneous tissue for primary closure. If there is insufficient skin, then tissue expanders, local tissue rearrangement, or distant flaps need to be considered. Second, if there is insufficient fascia, then component separation with primary fascial closure and mesh onlay or a mesh interposition are options for fascial closure. Since the fascial defect is 30 cm, it is unlikely that primary fascial closure can be achieved with component separation alone. An interposition of acellular dermal matrix is appropriate to bridge the fascial gap if primary fascial closure cannot be achieved.

2017

How well did you know this?
1
Not at all
2
3
4
5
Perfectly
21
Q

A 64-year-old man is brought to the emergency department after collapsing at home. Examination shows a ruptured abdominal aortic aneurysm and hemodynamic instability. The patient is taken to the operating room to undergo open repair of the aneurysm. Postoperatively, urine output decreases despite aggressive fluid resuscitation, and urinary bladder pressure is greater than 30 mmHg. Increased peak airway pressures are noted. Which of the following is the physiologic effect of increased intra-abdominal pressure in this patient?

A) Direct organ compression leads to decreased systemic afterload
B) Elevation of the diaphragm leads to decreased preload
C) Elevation of the diaphragm leads to increased flow in the inferior vena cava
D) Vascular compression leads to increased flow in the inferior vena cava
E) Vascular compression leads to decreased renal vascular resistance

A

The correct response is Option B.

Abdominal compartment syndrome (ACS) may develop rapidly after an increase in intra-abdominal pressure. Chronic causes of elevated intra-abdominal pressure (such as central obesity or large abdominal tumors) may be compensated for, but acute elevations as a result of trauma, bleeding, burn, or abdominal surgery may lead to life-threatening failure of multiple organ systems.

Three mechanisms present in ACS lead to multiple organ failure: vascular compression, elevation of the diaphragm, and direct organ compression. These three forces and their interactions create a constellation of physiologic effects that lead to the circulatory collapse at the center of ACS. Vascular compression results in decreased flow to the inferior vena cava and an increase in renal vascular resistance. Diaphragmatic elevation results in decreased flow to the inferior vena cava and an increase in intrathoracic pressure and decreased cardiac pre-load. Direct organ compression leads to an increase in systemic afterload. Left untreated, these forces eventually lead to renal failure, respiratory failure, intracranial hypertension, and intestinal and hepatic ischemia.

2017

How well did you know this?
1
Not at all
2
3
4
5
Perfectly
22
Q

An 83-year-old woman comes to the office for repair of a deep sternal wound infection after undergoing open single-vessel coronary artery bypass grafting using the left inferior mammary artery. Medical history includes cancer of the left breast for which she underwent wide excision with oncoplastic reconstruction followed by adjuvant radiation therapy to the breast and axilla. She does not smoke cigarettes. The patient’s infection resolves with aggressive debridement and culture-guided antibiotics. A photograph is shown. Which of the following is the most appropriate reconstructive option?

A) Left pectoralis turnover flap coverage with skin graft resurfacing
B) Left vertical rectus abdominis musculocutaneous flap coverage
C) Right latissimus dorsi musculocutaneous flap coverage
D) Right pectoralis advancement flap with skin readvancement closure
E) Right vertical rectus abdominis musculocutaneous flap coverage

A

The correct response is Option E.

Deep sternal wound infections represent life-threatening infections whose most frequent etiology in contemporary cases is surgical site infection following open heart surgery, with a reported incidence of 0.2 to 3% of patients undergoing median sternotomy. Risk factors in adults include diabetes mellitus, obesity, peripheral arterial disease, tobacco use, reoperation, and other surgical complications such as prolonged operative time and postoperative bleeding. Published mortality rates range from 8.1 to 14.8%. Once a diagnosis is made, initial treatment in cases of deep infection typically involve a combination of antimicrobial therapy and staged surgical debridement followed by delayed closure.

Sternotomy wounds can be complex problems because of their proximity to heart and lungs and exposure of these vital organs following debridement. Principles of reconstruction once a healthy wound bed has been obtained include durable coverage of vital structures with obliteration of dead space with a view to retaining or restoring optimal form and function.

In the case scenario described, a right superiorly based rectus abdominis myocutaneous flap provides the most reliable, well vascularized and non-irradiated option for regional soft tissue reconstruction of the composite chest wall defect (shown). A left vertical rectus abdominis myocutaneous flap is suboptimal in the setting of a relatively compromised superior epigastric vascular pedicle in the setting of known sacrifice of its parent left internal mammary artery (IMA). Latissimus and pectoralis flaps in any form are less reliable in their ability to fully cover inferior third sternectomy defects. While a turnover pectoralis flap can reach the inferior sternum, a left pectoralis turnover flap is furthermore less reliable in the setting of known sacrifice of its inferior segmental internal mammary perforators in the setting of known IMA sacrifice. Skin re-advancement in the setting of prior radiation and distorted perfusion from prior reduction mammoplasty also invites further unnecessary risk.

2017

How well did you know this?
1
Not at all
2
3
4
5
Perfectly
23
Q

A 55-year-old man who underwent abdominal surgery 10 years ago undergoes lysis of adhesions for treatment of ongoing intermittent bowel obstruction symptoms. He does not smoke cigarettes and has a history of hypertension and diabetes mellitus. A single enterotomy is made and repaired primarily. During abdominal wall closure after the intra-abdominal procedure, the fascial edges cannot be approximated without marked tension with a relaxed defect diameter maximum of 7 cm. Which of the following is the best method of repair?

A) Bioprosthetic mesh bridging
B) Component separation and bioprosthetic mesh underlay
C) Component separation and synthetic mesh underlay
D) Component separation with no mesh
E) Synthetic mesh bridging

A

The correct response is Option B.

Given the size of the defect, the patient’s comorbidities, and bowel violation, the best method to optimize results is a component separation with a bioprosthetic mesh underlay.

The use of bridging mesh without approximation of the fascia is not recommended due to a high recurrence rate. Instead the Ventral Hernia Working Group (VHWG) recommends reapproximation of the rectus muscle at the midline whenever possible without undue tension. This can be done by using the component separation technique originally described by Ramirez. The procedure calls for a release of the external oblique aponeurosis 1 cm lateral to the linea semilunaris, which allows for medialization of the rectus abdominis and underlying lateral musculature for primary approximation.

Given the enterotomy, a bioprosthetic mesh would be recommended as opposed to a synthetic mesh, because it is likely more resistant to infection and does not necessarily need removal in a contaminated wound. It is the VHWG’s preferred method to place mesh in an underlay manner, because intra-abdominal pressure pushes the mesh against the native abdominal wall instead of away from it. It also adds another layer of tissue over the prostatic material and would be preferred with a bioprosthetic because of decreased risk for bowel adhesions.

Additionally, lower rates of hernia recurrence have been shown in patients who underwent component separation with mesh as opposed to those without.

2016

How well did you know this?
1
Not at all
2
3
4
5
Perfectly
24
Q

A) Advancement of the vertical rectus abdominis musculocutaneous flaps
B) Release and advancement of the external oblique muscles
C) Release and advancement of the rectus, internal oblique, and transversus abdominis muscles
D) Rotation of the anteriolateral thigh flaps
E) Rotation of the transverse rectus abdominis myocutaneous flaps

A

The correct response is Option C.

A component separation procedure allows for the primary closure of central abdominal wall musculofascial defects without excessive tension. In the classically described procedure, the external oblique aponeurosis is released lateral to the linea semilunaris, allowing for medial advancement of the rectus abdominis along with the internal oblique and transversus muscles. The innervation and vasculature to these structures are preserved in order to maintain functional integrity of these abdominal wall muscles.

The vertical and transverse rectus abdominus musculocutaneous flaps along with the anterolateral thigh flaps are primarily used to reconstruct soft-tissue defects but are unlikely to result in a functional abdominal wall.

2016

How well did you know this?
1
Not at all
2
3
4
5
Perfectly
25
Q

A 66-year-old man with a history of adenocarcinoma of the lung undergoes a left pneumonectomy using a posterolateral thoracotomy incision. Postoperatively, he receives radiation therapy. The patient subsequently develops a nonhealing ulcer of the chest wall measuring 5 × 6 cm in the region of the nipple-areola complex. Which of the following is the most appropriate option for reconstruction?

A) Negative pressure wound therapy
B) Pedicled intercostal muscle flap and a split-thickness skin graft
C) Pedicled latissimus dorsi musculocutaneous flap
D) Pedicled vertical rectus abdominis musculocutaneous flap
E) Split-thickness skin graft only

A

The correct response is Option D.

Tissue injury from radiation results in irreversible damage that limits the ability of wounds to heal with skin grafts or by secondary intention, such as using negative pressure wound therapy. Tissues within the field of radiation are also generally affected, which would limit use of an intercostal muscle flap. Although a latissimus dorsi musculocutaneous flap would provide an adequate amount of tissue from outside of the field of radiation, it would be unreliable in the setting of prior posterolateral thoracotomy, unless there is documentation that the latissimus was spared. A vertical rectus abdominis musculocutaneous flap would provide sufficient tissue from outside of the field of radiation and would not have been affected by the patient’s prior surgery.

2016

How well did you know this?
1
Not at all
2
3
4
5
Perfectly
26
Q

A 24-year-old man is evaluated because of a 15-month history of painful idiopathic gynecomastia. Which of the following is the most appropriate treatment?

A) Anastrozole
B) Radiation therapy
C) Spironolactone
D) Surgical resection
E) Tamoxifen

A

The correct response is Option D.

Adult gynecomastia occurs because of drugs, unresolved pubertal gynecomastia, and unknown causes (idiopathic gynecomastia). Tamoxifen and raloxifene, both selective estrogen receptor modulators (SERMs), can be used for the treatment of male gynecomastia. Indeed, the use of SERMs is recommended to prevent gynecomastia as a result of antiandrogen monotherapy for treatment of prostate cancer. They are most effective, however, when used to treat gynecomastia that has been present for less than 1 year. Anastrozole, an aromatase inhibitor, is recommended for the treatment of breast cancer and as a preventative measure in high-risk women.

Surgical resection is the appropriate treatment for painful gynecomastia that has been present for greater than 1 year. Both open resection and liposuction are appropriate techniques.

Radiation therapy is indicated to prevent gynecomastia as a result of prostate cancer treatment.

Spironolactone is a cause of—not a treatment for—gynecomastia.

2016

How well did you know this?
1
Not at all
2
3
4
5
Perfectly
27
Q

An otherwise healthy 25-year-old man comes to the office with concerns about recent enlargement of his breasts. BMI is 28 kg/m2 and has not changed during the past year. He does not take any prescription or illicit drugs. He has no nipple discharge. Examination of which of the following is most appropriate?

A) Cervical lymph nodes
B) Cranial nerves
C) Peripheral pulses
D) Prostate
E) Scrotum and testes

A

The correct response is Option E.

The patient described is likely to have gynecomastia, which can occur in up to 30% of men. In the younger, postpubertal patients, testicular cancer may be a cause. Examination of the scrotum and testes to rule out any testicular masses has to be included in the physical examination and documented. Measurement of beta-human chorionic gonadotropin concentration can also be effective.

The other examinations listed do not directly contribute to the assessment of gynecomastia.

2016

How well did you know this?
1
Not at all
2
3
4
5
Perfectly
28
Q

Which of the following is the most appropriate term for the amount of air that is inspired and expired with a single breath during normal resting respiration?

A) Dead-space volume
B) Functional capacity
C) Residual volume
D) Tidal volume
E) Vital capacity

A

The correct response is Option D.

Tidal volume (VT) is the volume of air that is moved into or out of the lungs during quiet breathing. Tidal volume can be measured directly through spirometry or estimated based on a patient’s ideal body mass. It is a key parameter in mechanical ventilation to allow adequate ventilation without causing barotrauma to the lungs.

Vital capacity (VC) is the volume of air expired after deepest inspiration.

Functional capacity is a physiologic description of an individual’s ability to complete activities of daily living. It can be estimated through exercise treadmill testing and reported in metabolic equivalents (METs).

Residual volume (RV) is the volume of air remaining in the lungs after maximal exhalation.

Dead-space volume is the volume of air inhaled that does not take part in gas exchange. This volume can include both gas that remains in conducting airways (e.g., trachea, bronchi) during respiration and gas that reaches nonfunctional alveoli (e.g., nonperfused lung parenchyma following pulmonary embolism).

2016

How well did you know this?
1
Not at all
2
3
4
5
Perfectly
29
Q

A 27-year-old woman is evaluated for a recurrent abdominal desmoid tumor. CT scan shows a mass that occupies the full-thickness right musculofascial abdominal wall, involving the rectus abdominis muscle and oblique muscles, including lateral to the semilunar line. Resection is performed. Photographs of the defect are shown. Which of the following is the most appropriate management?

A) Bilateral component separation, primary skin closure with incisional topical negative pressure wound therapy, adjuvant chemotherapy
B) Left component separation, bridging wide intraperitoneal underlay biologic mesh, primary closure skin
C) Pedicled right anterolateral thigh flap with rectus femoris, no mesh
D) Placement of a bridging inlay of biologic mesh with primary split-thickness skin grafting and negative pressure wound therapy
E) Placement of a bridging inlay of uncoated heavyweight polypropylene mesh with adjuvant radiation therapy

A

The correct response is Option B.

The lesion in this patient is a recurrent desmoid tumor, also known as aggressive fibromatosis. It is a benign tumor, usually found in younger patients between 10 and 40 years of age, and is locally aggressive. It is often associated with pregnancy and previous surgery and can frequently recur. Management is en bloc, full-thickness, wide local excision (usually with frozen section confirmation of negative margins). An aggressive full-thickness abdominal wall resection is standard of care, making reconstruction more challenging.

The more durable and functional reconstruction entails complete restoration of the abdominal wall, especially musculofascial components, in a primary reapproximation. However, depending on the size of the tumor and resultant defect, this may not be possible. Basic principles, after obtaining proper margins after resection of the tumor, would then be reduction in defect size to the maximal extent possible and wide bridging underlay of mesh with at least 4- to 5-cm margins in all directions. Bridging inlay, whereby a mesh is simply sewn to the margins of the defect, has clearly been shown to be inferior in terms of recurrence rates.

Mesh choice can be either synthetic or biologic, although if significant contamination exists, if soft-tissue coverage is tenuous, or if one desires to decrease the amount of adhesion formation when placing mesh directly against the bowel, one should consider placement of biologic mesh, accepting the fact that there is a higher incidence of postoperative bulges using these materials, by and large.

In this case, only a left component separation is possible, given that the tumor has invaded the right rectus muscle and obliques, precluding their use for myofascial advancement.

There is no role for chemotherapy or radiation therapy in the treatment of these tumors.

Coverage with a right anterolateral thigh flap, with or without rectus femoris, can reconstruct the soft-tissue defect, but avoiding the use of mesh in a defect over 4 cm has a significantly higher chance of a recurrent hernia and would not be standard of care.

Primary skin grafting on top of a nonvascularized thick piece of acellular dermal matrix will not “take,” even with use of negative pressure wound therapy.

2015

(Please note that this pictorial appears in color in the online examination)

How well did you know this?
1
Not at all
2
3
4
5
Perfectly
30
Q

A 57-year-old woman undergoes resection of a squamous cell carcinoma of the vagina, resulting in a defect of the posterior two thirds of the vaginal vault from the introitus to the dome. A small portion of the rectum is involved in the resection and primarily repaired. The patient has a history of pelvic radiation therapy and tobacco use. Which of the following is the most appropriate approach for closure of this defect?

A) Bilateral gracilis muscle flaps
B) Deep inferior epigastric artery perforator flap
C) Primary repair of the vaginectomy defect
D) Split-thickness skin grafting with obturator
E) Vertical rectus abdominis musculocutaneous flap

A

The correct response is Option E.

In the clinical scenario described, the vertical rectus abdominis musculocutaneous (VRAM) flap is the most appropriate choice. This approach can provide enough tissue to resurface the vaginal vault and fill dead space.

For posterior defects of the vaginal vault, abdominal-based flaps are usually preferable. They can provide a large amount of vascularized tissue that rotates easily into the defect. Bilateral gracilis muscle flaps alone would offer less soft tissue, and mucosalization in a radiated field is unpredictable. A deep inferior epigastric artery perforator flap is far more complicated than a VRAM flap, and the donor site is problematic in patients with a history of smoking.

Primary closure of a large vaginal defect would likely cause significant stenosis and poor healing because of previous radiation. A skin graft with obturator is not likely to be successful given her radiation and tobacco history and violation of the rectum.

2015

How well did you know this?
1
Not at all
2
3
4
5
Perfectly
31
Q

A 62-year-old woman presents with a new-onset draining sinus of the left thoracic cage with associated indurated skin. Medical history includes bilateral breast cancer that was managed with bilateral radical mastectomy with radiation therapy 27 years ago. On CT scan, the image (shown) is consistent with osteoradionecrosis. Resection of affected skin, soft tissue, and thoracic cage produces a 35 × 20-cm soft-tissue defect and a skeletal defect spanning five ribs. A photograph of the defect is shown. The thoracic cage is fibrotic and noncompliant because of previous radiation. Which of the following approaches is most appropriate for reconstruction?

A) Free omental flap with skin graft over titanium mesh and reconstruction plates
B) Left latissimus dorsi muscle flap with skin graft over acellular dermal matrix
C) Left rectus abdominis turnover flap with skin graft over methyl methacrylate sandwich
D) Reverse abdominoplasty advancement over ePTFE patch
E) Right pectoralis muscle turnover flap over polypropylene mesh

A

The correct response is Option B.

(Please note that this pictorial appears in color in the online examination)

The most appropriate option for this patient is a left latissimus dorsi muscle flap with skin graft over acellular dermal matrix, given the alternatives listed. Basic principles of thoracic reconstruction include: debridement of devitalized tissue, removal of foreign bodies, establishment of healthy wound bed, restoration of stability/structure (generally reconstruction of skeleton if more than four ribs or a greater than 5-cm-diameter defect is involved), restoration of normal respiratory mechanics, protection of vital structures/organs, obliteration of dead space, provision of durable coverage, and delivery of an aesthetic result. However, if a patient has been previously irradiated, and therefore loses compliance of the thoracic cage because of radiation-induced fibrosis, skeletal reconstruction may not be mandatory if there is no paradoxical motion of the thoracic cage upon respirations and there is preservation of respiratory efficiency. Such is the case with this patient.

A left rectus turnover flap would not be a good option for two reasons: 1) as can be seen in the image, the left internal mammary artery has been harvested, thereby compromising the superior epigastric vessel on which this flap would be based, and 2) it is insufficient to provide enough soft-tissue coverage of a defect this size. Furthermore, as indicated above, methyl methacrylate would not be mandatory in this patient.

A right pectoralis turnover flap is insufficient to cover a defect this size.

A free omental flap can be used to reconstruct this defect (as can a pedicled omental flap), but again, thoracic skeletal reconstruction would not be mandatory in this previously irradiated patient; furthermore, even if it were, titanium mesh and reconstruction plates would not be utilized.

A reverse abdominoplasty flap (Ryan procedure) would not be able to cover a defect this size.

How well did you know this?
1
Not at all
2
3
4
5
Perfectly
32
Q

A 56-year-old man is evaluated for a ventral hernia after undergoing midline laparotomy for diverticulitis. BMI is 38 kg/m2. Physical examination shows midline fascial defect measuring 20 × 15 cm; there is no evidence of infection and skin coverage is stable. Repair with rectus advancement and polypropylene mesh is planned. Placement of mesh between which of the following planes is most likely to decrease this patient’s risk of hernia recurrence?

A) Anterior rectus sheath and rectus muscle
B) Internal and external oblique muscles
C) Medial edges of rectus muscle
D) Rectus muscle and posterior sheath
E) Skin and anterior rectus sheath

A

The correct response is Option D.

Although recurrence rates are generally very high for large ventral hernias, placement of mesh in the retrorectal position appears to have the most decreased rate of recurrence compared with other methods. Placement above the plane of the rectus muscle requires division of vascular perforators that traverse the rectus muscle and perfuse the overlying skin flaps. These perforating branches of the epigastric circulation are most dense in the periumbilical zone. Preservation of the perforators has been shown to be beneficial in a number of case series reports. Placement of mesh between the oblique muscles in this case would not provide support for the midline hernia because these muscles are more lateral.

Hernia recurrence rates are generally more increased in the presence of infection, with large defects when the rectus muscles cannot be advanced back together in the midline, in obese patients, and in patients with multiple medical comorbidities. Patients with several risk factors can expect recurrence rates in the range of 20 to 40%, whereas patients with few risk factors have recurrence about 5% of the time.

2015

How well did you know this?
1
Not at all
2
3
4
5
Perfectly
33
Q

A 60-year-old man undergoes sigmoid resection and colostomy for management of ruptured diverticulitis. The patient has smoked one pack of cigarettes daily for the past 35 years. BMI is 36 kg/m2. After colostomy reversal, he has an abdominal wound infection and fascial dehiscence. Reconstruction with a bridging human acellular dermal matrix is planned. Compared with traditional polypropylene mesh repair, which of the following complications is more likely with the planned approach?

A) Abdominal bulge
B) Fistula
C) Hematoma
D) Infection
E) Skin necrosis

A

The correct response is Option A.

Acellular dermal matrices (ADM) have been advocated for the past decade as an important adjunct in the complex field of abdominal wall reconstruction. Many studies have verified the use and general safety of ADM in abdominal wall reconstruction, but conclusive evidence of its advantages over other techniques is still lacking. What can be inferred is its advantage over prosthetic mesh in contaminated fields. Polypropylene mesh would be contraindicated in the infected wound in this example. After adequate debridement and appropriate antibiotics in an optimized patient, ADM can be used for hernia repair or reconstruction along with component separation. Postoperative infections can be as common as 40%, but conservative management measures often suffice, rather than reoperation and graft explantation, which are required with prosthetic mesh.

One recognized drawback in the stretchable nature of dermal matrix grafts is that they can often stretch under tension to 50% or more of their initial dimensions. Postoperative bulging without true herniation is common. Strategies for prevention include suturing the graft under maximal stretch and use of porcine dermal grafts rather than human grafts.

Hernia recurrence in the complex abdominal wall reconstruction remains a common event, regardless of technique. Although some studies assert a strong advantage with ADM, others report a similar or increased recurrence rate. As successful operative techniques become more standardized, perhaps more uniform success will be demonstrated in future studies.

Skin necrosis and hematoma are common surgical complications that should not vary between choice of graft material.

Fistula rates are decreased with ADM versus prosthetic mesh reconstruction.

2015

How well did you know this?
1
Not at all
2
3
4
5
Perfectly
34
Q

A 40-year-old man undergoes ventral hernia repair with biologic mesh and fascial closure at the midline. A bilateral component separation technique with incision of the external oblique fascia and muscle lateral to the linea semilunaris and dissection in the plane between the external and internal oblique muscles, and separation of the rectus muscle off of the posterior rectus fascia is performed. At which of the following levels can the least amount of advancement of the medial fascial edges be expected?

A) Midway between the umbilicus and pubis
B) Midway between the umbilicus and subcostal margin
C) Subcostal margin
D) Suprapubic
E) Umbilicus

A

The correct response is Option C.

The component separation technique can be used to achieve medial transposition of the rectus muscle and overlying anterior fascia. The surgery involves division of the external oblique fascia and muscle lateral and parallel to the linea semilunaris. The plane deep to the external oblique muscle, which is relatively avascular, is then dissected laterally. The rectus muscle is also separated off of the posterior rectus sheath (using access from the medial laparotomy or hernia incision). This allows for medial advancement of the rectus muscle, overlying anterior rectus sheath, internal oblique muscle, and transversus muscle as a unit. The segmental neurovascular bundles course deep to the internal oblique muscle and penetrate into the rectus muscle 10 to 25 mm medial to its lateral margin.

The component separation technique, when performed in the scenario described, can give unilateral advancement toward the midline approximately 10 cm at the level of the umbilicus, which equates to a bilateral advancement of 20 cm. The least amount of advancement is in the subxiphoid and subcostal regions, often making more cranially located defects more difficult to close. Since it was originally reported in 1990, several modifications and variations of this technique have been described in the literature. These include perforator-preserving and/or endoscopic techniques to methods that describe additional maneuvers to increase mobilization or improve durability with the addition of biologic or prosthetic meshes.

2015

How well did you know this?
1
Not at all
2
3
4
5
Perfectly
35
Q

A 45-year-old woman comes to the office seeking consultation for an abdominoplasty. Physical examination shows diastasis recti and excess skin and fat in the infraumbilical region. Abdominoplasty with undermining of the upper abdominal flap and plication of the diastasis is planned. Decrease in overall sensation in which of the following areas is most likely after more than 3 years postoperatively?

A) Epigastric
B) Infraumbilical
C) Lateral abdominal
D) Subxiphoid
E) Suprapubic

A

The correct response is Option B.

The area below the umbilicus and above the incision is most likely to have decreased sensation in the long term (after more than 3 years). This is true for superficial touch, superficial pain, temperature (hot and cold), and vibration. The area above the umbilicus recovers sensation to touch, pain, and vibration within 3 years; sensation to hot and cold temperatures recovers beyond 3 years. The suprapubic area has decreased sensation to temperature; however, touch, pain, and vibration recover in the short term. The areas below the xiphoid, at the lateral abdomen, as well as all other areas on the abdomen, have decreased sensation to pressure, although the infraumbilical region had the greatest change.

The anterior nerve branches of the 6th to 12th intercostal nerves travel in a plane between the transversus abdominis and internal oblique muscles. They enter the internal oblique fascia, divide into two branches, and enter the posterior rectus sheath. One branch ascends 3 cm from the lateral edge of the rectus muscle to supply sensation to the skin over the lateral half of the rectus muscle. The other branch travels between the rectus and the posterior sheath before penetrating the linea alba and supplying sensation to the medial half of the rectus muscle. These nerves are divided during undermining from the abdominoplasty.

2015

36
Q

A 65-year-old man is evaluated for a large mass in a previously irradiated area of the posterolateral chest. Preoperatively obtained ventilation-perfusion scans and pulmonary function testing are within normal limits. A full-thickness chest wall resection to include three ribs and at least 10 cm of each rib is planned. The soft-tissue defect is anticipated to be 20 cm in diameter. Which of the following factors in this patient is most likely to necessitate a rigid chest wall reconstruction?

A) Posterolateral position of the defect
B) Previous chest wall radiation therapy
C) Total area of the chest wall defect
D) Total number of ribs resected

A

The correct response is Option C.

The most likely factor to indicate rigid chest wall reconstruction in this very large anticipated chest wall defect is the total area of the bony chest wall that is resected. Generally, reconstruction of the lateral bony chest wall seems necessary if four or more consecutive ribs are resected or if the diameter of the total defect is larger than 5 cm.

Historically, protection of a flail segment of chest wall was based on the pendelluft principle, a phenomenon in which there is airflow back and forth between the lungs, resulting in underventilated segments of lung. This out-of-phase movement of the airway gas between the intact and flail-chest-side lungs was long believed to be the major contributor to respiratory dysfunction in patients with flail chest but has failed to be proven and appears to be a flawed hypothesis.

In patients who have had radiation therapy, larger defects may be tolerated without rigid chest wall stabilization owing to fibrosis. As many as five ribs may be resected in patients who have undergone radiation therapy before reconstruction is considered because increased fibrosis produces chest wall stiffness.

Anterior and posterior defects are typically better tolerated than lateral defects.

2015

37
Q

Which of the following represents the lateral border of the breast footprint?

A) Anterior axillary line
B) Anterior edge of the latissimus dorsi muscle
C) Lateral clavicle
D) Midaxillary line
E) Posterior axillary line

A

The correct response is Option A.

The lateral breast is mobile, unlike the inframammary fold. Therefore, the footprint can vary slightly and be just behind the anterior axillary fold, but it does not extend to the midaxillary line. The breast footprint is not described as relating to either the latissimus dorsi or the clavicle.

2015

38
Q

A 43-year-old woman brings her 10-day-old daughter for evaluation because she is concerned about the appearance of her daughter’s nipples. The newborn was born at full-term after an uncomplicated pregnancy. Physical examination shows that the newborn has inverted nipples bilaterally. Which of the following is the most appropriate next step in management?

A) Continuous elastic outside distraction starting at day of life 14
B) Suction lengthening of the nipple starting at day of life 14
C) Suction lengthening of the nipple starting immediately
D) Transection of the fibrotic bands at 6 months of age
E) Observation

A

The correct response is Option E.

It is very common for nipples to be inverted at birth and concerned parents often discuss the condition with pediatricians and practitioners. The mammary ridge begins to develop in the fourth week of development, and produces the primary and secondary buds. The mammary pit is a small depression present at birth into which the lactiferous ducts open. After birth, proliferating mesoderm during the first several weeks of life will cause the inverted nipple to protrude.

In some cases, the inverted nipple persists. Although there may not be a functional consequence, it can lead to difficulty with lactation. Han and Hong described a grading system for inverted nipple: grade I, in which the nipple can be made to protrude manually and can maintain position without traction; grade II, in which the nipple fails to maintain projection after manipulation; and grade III, in which the nipple cannot be pulled out manually. Treatment is based on the grade. In some cases, breast-feeding or use of a breast pump is sufficient to correct the variation. Surgical intervention can be approached in several ways. Constricting the base of the inverted nipple can create projection, severing the ducts can cause projection, and myotomy can cause projection. Any division of the ducts will compromise a patient’s ability to breast-feed.

2015

39
Q

Gynecomastia that occurs in men who take anabolic steroids is caused by increased levels of which of the following?

A) Aromatase
B) Estradiol
C) High-density lipoprotein
D) Testosterone

A

The correct response is Option B.

Anabolic steroids have sex-specific adverse effects. Development of breast tissue in men, a condition called gynecomastia (which is usually caused by high levels of circulating estradiol), may arise because of increased conversion of testosterone to estradiol by the enzyme aromatase. Decreased sexual function and temporary infertility can also occur in men.

2015

40
Q

A 16-year-old girl comes to the clinic because she is dissatisfied with the asymmetric appearance of her breasts. Physical examination shows the right nipple-areola complex is more superiorly located and the breast volume is small. There is absence of the right anterior axillary fold. Which of the following other physical examination findings is most likely?

A) Accessory nipple
B) Microtia
C) Right clubfoot
D) Scars consistent with repaired cleft lip
E) Shortened right-hand digits

A

The correct response is Option E.

The chest findings described are consistent with Poland syndrome with absence of the sternocostal head of the pectoralis major muscle. Poland syndrome can be associated with hand abnormalities, including shortened digits. Cleft lip, accessory nipple, clubfoot, and microtia are not known to be associated with Poland syndrome.

2015

41
Q

A 19-year-old man with no available medical history presents for initial evaluation of gynecomastia. Which of the following is most appropriate to include in this patient’s workup?

A) Chest x-ray study
B) Hand x-ray study
C) Mammography
D) Testicular examination

A

The correct response is Option D.

Gynecomastia can occur because of imbalances of estrogen and testosterone, and it can be associated with obesity, certain drugs, and Klinefelter syndrome. It can also be associated with testicular tumors. Therefore, a testicular examination should be performed. Mammography is not necessary for a routine workup if physical examination shows no abnormal masses. A chest x-ray study is not needed for an otherwise healthy 19-year-old. An x-ray study of the hand can evaluate closure of growth plates when determining if puberty has completed. This is unlikely to be necessary for someone who is 19 years old.

2015

42
Q

The dominant vascular supply of the rectus abdominis muscle originates from which of the following vessels?

A) Common femoral
B) External iliac
C) Internal iliac
D) Internal mammary
E) Superficial femoral

A

The correct response is Option B.

Component separation for closure of large abdominal wall defects was first described by Ramirez in 1990. The purpose of the surgery is to achieve abdominal wall closure with well-vascularized, innervated muscle flaps. The primary vascular supply to the rectus muscles are the deep inferior epigastric artery and vein, which arise from the external iliac vessels.

The internal mammary vessels give rise to the superior epigastric arteries and veins, which is a secondary, nondominant vascular supply of the rectus muscles. The femoral vessels give rise to the superficial inferior epigastric artery and vein, which perfuse the skin and subcutaneous fat of the inferior lateral abdomen.

2015

43
Q

A 62-year-old man is evaluated for an 8-month history of fullness of the right breast associated with a subareolar mass. He reports no history of pain, nipple discharge, skin changes, or systemic symptoms. There is no family history of breast cancer. Examination of a specimen obtained on biopsy shows ductal epithelial hyperplasia with proliferation of stroma and fibroblasts. Which of the following is the most likely diagnosis?

A) Ductal carcinoma
B) Epidermal cyst
C) Fibroadenoma
D) Gynecomastia
E) Lymphangioma

A

The correct response is Option D.

Gynecomastia is defined as the benign proliferation of male glandular breast tissue. The most common symptom of gynecomastia is painless breast enlargement in adolescent and elderly men. Examination of a breast biopsy specimen may be necessary to exclude breast cancer if mammography and/or breast ultrasonography are suggestive of malignancy. Microscopic findings include ductal epithelial hyperplasia with proliferation of stroma and fibroblasts. Pseudogynecomastia is defined as breast enlargement secondary to fat deposition without glandular proliferation. Fibroadenoma are the most common benign tumor of the female breast. They are most often diagnosed in women between the ages of 20 and 35 years. Fibroadenomas are often ovoid or spherical, freely movable, and often well circumscribed. Microscopic findings include ductal tissue with proliferation of stroma surrounded by fibroblasts. Ductal carcinoma accounts for the majority of breast cancers. Ductal carcinoma is characterized microscopically by cords and nests of tumor cells with varying amounts of gland formation, and cytologic features that range from bland to highly malignant. Epidermal cysts are inclusion cysts lined by well-differentiated epidermis filled with keratin. Lymphangiomas of the breast are distended lymphatic channels interspersed with breast lobules.

2015

44
Q

A 29-year-old woman comes to the office because of a firm, mildly tender, well-circumscribed mass of the abdomen. A photograph is shown. The mass has been slowly increasing in size for the past 6 months. CT scan shows a mass that occupies the left musculofascial abdominal wall, including the rectus, external, and internal oblique muscles, and penetrates through the anterior rectus sheath. Which of the following is the most appropriate management?

A) Neoadjuvant chemotherapy
B) Radiation therapy
C) Wide local excision, bilateral component separation, adjuvant chemotherapy
D) Wide local excision, mesh placement, radiation therapy
E) Wide local excision, right component separation, mesh reinforcement

A

The correct response is Option E.

The lesion in this patient is a desmoid tumor, also known as aggressive fibromatosis. It is a benign tumor, usually found in younger patients between 10 and 40 years old, and is locally aggressive. It is oftentimes associated with pregnancy and prior surgery, and can frequently recur. Treatment is en bloc full-thickness wide local excision (usually with frozen section confirmation of negative margins). As aggressive full-thickness abdominal wall resection is standard of care, reconstruction is more challenging.

If midline fascia can be reapproximated, it should be, as primary fascial closure is associated with the lowest hernia recurrence rates. Reinforcement with mesh has been prospectively demonstrated to reduce recurrence rates even further, especially in defects over 4 cm. If midline fascial reapproximation is not possible, reduction in the size of the defect is crucial to decrease recurrence rates. This is done by component separation. However, in this case, only a right component separation is possible, given that the tumor has invaded the left rectus muscle and obliques, precluding their use for myofascial advancement. If the obliques had been spared, a component separation could still have been attempted even if there were violation of the rectus. The most durable reconstruction would be achieved if midline fascial reapproximation were possible with mesh reinforcement. Second best would be reduction in the size of the defect with a right component separation and placement of mesh as a bridging underlay.

There is no role for neoadjuvant chemotherapy or radiation therapy in the treatment of these tumors.

2014

45
Q

A 21-year-old woman comes to the office because of a lump 2 cm below the inframammary fold. She says she has “always had it,” but it grew larger during pregnancy and has not decreased. The mass was painful during breast-feeding, and it is occasionally tender. Which of the following is the most likely diagnosis?

A) Epidermal inclusion cyst
B) Lipoma
C) Polymastia
D) Polythelia
E) Sarcoma

A

The correct response is Option C.

Patients often come to plastic surgeons with subcutaneous masses. An understanding of the differential diagnosis is helpful to counsel patients. Accessory breast tissue (polymastia) along the milk line is common. This breast tissue is responsive to hormonal influence, and patients will describe cyclical pain and swelling coinciding with menses and with milk letdown. Surgical excision can be performed. Polythelia is the presence of accessory nipples. Lipomas, epidermal inclusion cysts, and sarcomas can occur as subcutaneous masses, but they do not change in character based on hormonal influence.

2014

46
Q

An otherwise healthy 27-year-old man comes to the office for removal of asymptomatic enlarged breasts that have persisted unchanged since onset at age 14. He takes no medications and does not use recreational drugs. Physical examination shows symmetrical collections of rubbery, firm subareolar tissue 4.5 cm in diameter. There is scant surrounding fatty tissue and no ptosis; areolas are 28 mm in diameter. Which of the following is the most appropriate management of this patient’s condition?

A) Circumferential periareolar resection with liposuction
B) Mastectomy via Wise pattern incision
C) Observation to allow for involution
D) Oral hormone-blocking medication
E) Subareolar tissue ressection via pull-through excision technique

A

The correct response is Option E.

Gynecomastia, enlargement of the male breast, may occur as physiologic temporary overgrowth of the adolescent breast or it may appear during adulthood as a result of numerous etiologies. It can consist of various proportions of excess subareolar fibrous breast tissue and adipose tissue in the periphery, with the extent often depending upon the habitus of the individual. Appearing at about age 14 in more than 65% of healthy boys, gynecomastia will typically resolve within 2 years of onset in otherwise healthy males. Persistence of adolescent-onset gynecomastia after age 21 is unlikely to resolve with conservative measures. Hormone blockers have no place in the management of persistent breast tissue in otherwise healthy individuals. A wide variety of surgical treatment options for resection of redundant male breast tissue have been published. These include direct excision through a number of described incisions, traditional and ultrasound-assisted liposuction, and recently, use of an arthoscopic shaver. Ultrasound-assisted liposuction may have an advantage in stimulating skin retraction for cases where mild to moderate ptosis exists when scar minimization is desired. For otherwise uncomplicated gynecomastia, excision of fragmented subareolar tissue via a limited areolar border incision with the pull-through technique is the most appropriate choice of therapy. When redundant skin exists in addition to excess male breast tissue, immediate skin resection via periareolar, transverse lenticular, omega-shaped, or Wise (inferior pedicle) incision are among the numerous methods described. Regardless of the resection method, an evolving trend favors allowing skin to retract for 6 to 9 months before determining whether there is a need to subject the individual to additional incisional scarring.

2014

47
Q

During fetal development, failure of the mammary pit to elevate above the skin level results in which of the following deformities?

A) Accessory nipple
B) Amastia
C) Amazia
D) Athelia
E) Nipple inversion

A

The correct response is Option E.

The congenitally inverted nipple is common and occurs in 4% of infants. It results from the failure of the mesenchyme to proliferate above the level of the skin.

Amastia is absence of the entire breast. Athelia is absence of the nipple. Amazia is absence of the mammary gland with an intact nipple and areola. Polythelia, or accessory nipple, results from failure of regression along the milk line.

2014

48
Q

A 5-year-old male has a cerebrospinal fluid leak and a 3 x 3-cm area of wound dehiscence involving the posterior trunk following tethered cord repair. Which of the following is the most appropriate method to reconstruct the wound?

A) Gluteal muscle flap and skin advancement flap
B) Latissimus muscle turnover flap and skin advancement flap
C) Local fascial flap and skin advancement flap
D) Skin advancement flap
E) Split-thickness skin graft

A

The correct response is Option C.

The most appropriate method to reconstruct the wound is a local fascial flap and skin advancement flap. The major principle of tethered cord and myelomeningocele repair is to obtain a well-vascularized layer of soft tissue coverage between the dural and skin closures. The fascia overlying the paraspinous muscles can be turned over as flaps to cover the underlying dural repair. This vascularized soft tissue layer will minimize the risk of cerebrospinal fluid leak by reinforcing the dural repair. In addition, the fascial flaps will prevent contact with cutaneous bacteria and subsequent meningitis if either the dural repair or skin repair breaks down. A split-thickness skin graft over the dura would not adequately protect the spinal cord. Closing the skin directly over the dural repair using skin advancement flaps would place the child at risk for meningitis in the event of a cerebrospinal fluid leak or if wound breakdown occurred along the incision line of the widely undermined skin flaps. The use of a regional gluteal or latissimus muscle flap to cover the dural repair is unnecessary because local tissue (paraspinous muscle fascia) is available. Harvesting the gluteal or latissimus muscles also may cause significant donor site morbidity in a child already at risk for ambulatory problems from a neurological deficit.

2019

49
Q

A 58-year-old man comes to the office with recurrent rectal cancer four years after undergoing low anterior resection with adjuvant radiotherapy. Open abdominoperineal resection is planned. Which of the following closure methods will most reliably reduce the incidence of perineal wound complications?

A) Gracilis flap closure
B) Negative pressure wound therapy
C) Primary closure
D) Rectus abdominis flap closure
E) Split-thickness skin grafting

A

The correct response is Option D.

Several retrospective studies and one randomized trial have shown that when compared to primary closure, the rectus abdominis myocutaneous flap reduces wound healing complications after abdominoperineal resection (APR). Gluteal and gracilis flaps have been used for reconstruction after APR; however, the data supporting their use is not as robust. Split-thickness skin grafting and negative pressure wound therapy are inappropriate for reconstruction of the APR due to the size of the wounds and the risk of evisceration.

2019

50
Q

A 65-year-old man presents with an infection of the sternum following aortic valve repair. After sternal debridement, there is a 10-cm-wide, deep wound from the clavicle to the upper abdomen. Which of the following is the most appropriate flap to reconstruct the wound?

A) Latissimus dorsi
B) Omentum
C) Pectoralis major
D) Pectoralis minor
E) Serratus

A

The correct response is Option B.

The most appropriate flap to reconstruct the wound is omentum. Because of the large extent of the wound, the only flap listed that can adequately fill the defect and eliminate the dead space is the omentum. Pectoralis major flaps would not adequately fill the defect, particularly the inferior aspect of the wound. Pectoralis minor flaps are not used for sternal reconstruction and would not provide adequate tissue. The latissimus dorsi flap would not be able to fill the large sternal wound. Serratus flaps can be used for posterior chest wounds, but would not be able to reconstruct the large anterior chest wound.

2019

51
Q

A 75-year-old woman with a history of right mastectomy and irradiation therapy presents with a sarcoma that requires radical resection and partial sternectomy. A photograph is shown. A pedicled flap is planned to repair the defect. Which of the following arteries supplies the most appropriate flap in this situation?

A) Deep inferior epigastric
B) Internal mammary
C) Lateral thoracic
D) Thoracoacromial
E) Thoracodorsal

A

The correct response is Option E.

The most appropriate pedicled flap for this particular defect is a latissimus flap, shown in the photograph, supplied by the thoracodorsal artery. The latissimus flap is a Mathes/Nahai type V flap that can be transferred on its dominant pedicle (thoracodorsal artery) or on multiple segmental paraspinal perforators. The internal mammary artery terminates as the superior epigastric artery, which would provide blood supply for a superiorly based vertical rectus flap. The rectus flap is a Mathes/Nahai type III flap, with two dominant pedicles. However, this pedicle is not available because of the radical resection and prior irradiation. The lateral thoracic artery is one of two dominant pedicles supplying the serratus anterior muscle (Mathes/Nahai type III). This flap is an option, but it would not provide enough bulk necessary for the defect in this situation. The deep inferior epigastric artery (DIEA) supplies the DIEA perforator flap, which would be an option as a free tissue transfer but not as a pedicle flap for this situation.

52
Q

A 52-year-old woman with cancer of the right breast undergoes mastectomy and axillary node dissection, complicated by mastectomy flap necrosis requiring skin grafting. She completes adjuvant chemoradiation. One year later, she comes to the office with a fungating mass growing through the skin graft. Imaging demonstrates involvement of the fourth and fifth ribs with an anticipated skeletal defect of 4 × 4 cm. A photograph is shown. Which of the following is the most appropriate treatment for the skeletal reconstruction?

A) High-density porous polyethylene
B) Methyl methacrylate with mesh
C) 2.4-mm Titanium plate
D) No skeletal reconstruction
E) Vascularized rib

A

The correct response is Option D.

The principles of management of this recurrent right breast cancer include radical resection of all involved tissues (including ribs) and reconstruction with well vascularized flaps. In this case, a right latissimus muscle flap and skin graft was used for reconstruction. No alloplastic material was placed or skeletal thoracic cage reconstruction performed. This is common in these types of patients, because excessive fibrosis caused by the radiation to the chest wall prevents loss of respiratory efficiency through paradoxical motion which otherwise occurs in patients who have more than four ribs involved or a defect larger than 5 cm.

2019

53
Q

A 56-year-old man is evaluated because of gynecomastia. Physical examination shows mild, diffuse breast enlargement with no visible inframammary fold or ptosis. Which of the following is the most appropriate surgical correction of this patient’s condition?

A) En bloc resection of skin and breast tissue with free nipple grafting
B) Open excision of breast tissue with mastopexy
C) Subcutaneous mastectomy with nipple preservation
D) Suction-assisted lipectomy
E) Superior periareolar excision with skin excision

A

The correct response is Option D.

The treatment of gynecomastia is based on the degree of breast enlargement and the extent of ptosis that is noted on examination. Grade 1 gynecomastia is minimal breast hypertrophy without ptosis. Grade II gynecomastia is moderate hypertrophy without ptosis. Grade III gynecomastia is severe hypertrophy with moderate ptosis. Grade IV gynecomastia is severe hypertrophy with severe ptosis. The treatment of mild to moderate gynecomastia without ptosis is suction-assisted lipectomy. Direct periareolar excision with skin excision and subcutaneous mastectomy are not indicated for gynecomastia without ptosis. Mastopexy and free nipple grafting techniques are indicated for gynecomastia with severe ptosis.

2019

54
Q

The postoperative CT scan shown is obtained to evaluate a wound dehiscence in a patient who underwent left-sided unilateral reduction mammaplasty for asymmetry six weeks ago. Which of the following upper extremity deformities is most likely to be found in this patient?

A) Contralateral radial club hand
B) Contralateral “pouce flottant” thumb
C) Ipsilateral brachysyndactyly
D) Ipsilateral radial head subluxation
E) Ipsilateral type IV Wassel preaxial polydactyly

A

The correct response is Option C.

Assessment of the chest CT shows right-sided absence of the pectoralis major and minor muscles and breast hypoplasia. The patient suffers from Poland syndrome, which is a congenital disorder of unknown etiology with the prevailing theory being hypoplasia of the subclavian artery or its branches during the sixth week of embryogenesis. Variability exists in physical findings with the most common being: anterior axillary fold and pectoralis major sternal head absence, breast gland thinning, rib and cartilage hypoplasia, and ipsilateral brachysyndactyly. After local wound care and antibiotic therapy, the patient had resolution of her symptoms.

A type IV Wassel preaxial polydactyly is the most common congenital thumb duplication but is not associated with Poland syndrome. Radial club hand is more common than ulnar club hand, but has no association with Poland syndrome. Both are congenital hand deformities, but unrelated to the pathological condition mentioned. Radial head subluxation is also known as “nursemaid’s elbow.” Nursemaid’s elbow is a common injury of early childhood. It is sometimes referred to as “pulled elbow” because it occurs when a child’s elbow is pulled and partially dislocates. There is no connection between Poland syndrome and increased incidence of radial head subluxation. Type IV Manske modification of the Blauth classification thumbs (the “pouce flottant” thumb) have rudimentary elements and are attached to the hand by a small skin bridge. These thumb anomalies are not associated with Poland syndrome.

2019

55
Q

A newborn male infant who is born at 36 weeks’ gestation via cesarean delivery has a large defect of the anterior abdominal wall. Examination shows matted bowel loops coming through the defect lateral to the umbilical cord. No other abnormalities are noted. Which of the following associated findings is/are most likely?

A) Abnormal karyotype
B) Constriction rings with limb and digital amputations
C) Elevated maternal serum alpha fetoprotein (MSAFP)
D) Hypoglycemia, macrosomia, and macroglossia
E) Translucent membrane covering bowel

A

The correct response is Option C.

Omphalocele (OC) and gastroschisis (GS) represent the two most common congenital abdominal wall defects, with a prevalence of approximately 3 to 4 per 10,000 live births/fetal deaths/stillbirths/pregnancy terminations each. Precise pathoetiologies are unclear, but developmental pathways and characteristics at the time of birth are notably distinct. OC is characteristically a midline partial-thickness abdominal wall defect covered by a membrane of amnion and peritoneum occurring within the umbilical ring and containing abdominal contents. GS is characteristically a full-thickness, paraumbilical abdominal wall defect associated with eviscerated bowel.

Both OC and GS are associated with elevated maternal serum alpha fetoprotein (MSAFP). For comparison, MSAFP values average twice that recorded in pregnancies with open spina bifida, and similar to values recorded with anencephaly. An elevated MSAFP is an indication for thorough ultrasound examination of the fetus for anatomical abnormalities.

Multiple chromosomal abnormalities have been associated with at least 60% OC cases, including trisomy -18, -13, -21, Turner syndrome, and triploidy. By contrast, GS is associated with abnormal karyotype in about 1% of cases, usually in the setting of other congenital abnormalities.

The definite treatment of both OC and GS is surgical once optimal resuscitation is achieved. Primary closure is associated with better survival rates if it can be achieved without compromise of intestinal blood flow or other hemodynamic or respiratory embarrassment. Large defects are frequently managed with temporary abdominal silos which are gradually reduced over the course of days to weeks in a form of visceral tissue expansion followed by delayed abdominal wall closure. The long-term outcome in isolated cases of OC and GS are generally good, although they can be associated with gut motility impairment, gastroesophageal reflux, ventral hernias, and late obstructive episodes.

Constriction rings with limb and digital amputations are found in amniotic band sequence but are not characteristic of OC or GS. GS is not characteristically associated with hypoglycemia, macrosomia, or macroglossia.

2019

56
Q

A 59-year-old woman presents with an infected sternal nonunion after coronary artery bypass grafting 4 weeks ago. After debridement of the wound, five sternal plates and bilateral pectoralis flaps are placed. Postoperatively, the patient becomes hypotensive, tachycardic, and confused. Jugular distention is noted. Oxygen saturation is 100% on nasal cannula. Which of the following is the most appropriate initial step in management?

A) Auscultation
B) Chest x-ray
C) ECG
D) Ultrasonography of the heart
E) Return the patient to the operating room

A

The correct response is Option A.

On auscultation a muffled heart sound and pericardial friction rub is heard and would direct the clinician to decompress tamponade.

Patient is demonstrating Beck’s triad and has reason for possible cardiac tamponade.

Immediate chest x-ray can be ordered to help rule out pneumothorax, but with normal oxygenation, the chance of a pneumothorax is lower on the differential, and there are other better initial diagnostic and therapeutic steps.

ECG can help support the diagnosis of pericardial effusion, but this is not diagnostic and is only used as an adjunct.

Ultrasonography of the heart can confirm the existence of pericardial effusion, as well as allow needle drainage for immediate treatment. However, this would be performed after auscultation.

2019

57
Q

A 55-year-old man presents for a large abdominal midline hernia repair. A component separation is planned with a posterior approach and a retrorectus mesh placement. Which of the following layers can be divided to provide further release and preserve the innervation to the rectus muscle?

A) Anterior rectus sheath
B) External oblique
C) Internal oblique
D) Transversalis fascia
E) Transversus abdominis

A

The correct response is Option E.

In the posterior component separation approach for ventral hernia repair, transversus abdominis release (TAR) can provide further mobility and preserve the innervation to the rectus muscle. The posterior approach reinforces hernia repair with a sublay mesh placed between the rectus muscle and posterior sheath. The Rives-Stoppa approach is associated with a 3 to 6% recurrence rate. To avoid disruption of the segmental nerves to the rectus, classical dissection was limited medial to the linea semilunaris. This, however, limited the space and reserved this technique for small- to medium-sized hernias. To extend this dissection laterally for use in larger defects, either the internal oblique or the transversus abdominis muscle can be divided. Division of the internal oblique divides the nerves to the rectus muscle. Division of the transversus abdominis can preserve these nerves. With this technique, the anterior rectus sheath is preserved as well as the external oblique and transversalis fascia.

2020

58
Q

A 65-year-old woman presents to the office with an ulcer on the right chest wall. She underwent right-sided mastectomy and adjuvant external beam radiation therapy for advanced breast cancer 5 years ago. Physical examination shows a 2-cm ulcer with surrounding radiation-damaged skin and no signs of acute infection. Which of the following is the most appropriate next step in management?

A) Biopsy of the wound
B) Excision of all radiation-damaged tissue and coverage with vascularized tissue
C) Excision of the ulcer and coverage with vascularized tissue
D) Hyperbaric oxygen therapy
E) Negative pressure therapy

A

The correct response is Option A.

Radiation causes production of reactive oxygen species, which causes injury to tissues and progenitor cells. Cytokine release results in chronic inflammation and ongoing tissue damage. Radiation therapy can cause soft-tissue ulcerations, osteoradionecrosis, and radiation-induced sarcomas. If a patient presents with a late ulcer after radiotherapy, malignancy needs to be ruled out. A biopsy of the ulcer edge should be performed.

Once malignancy has been ruled out, excision of all radiation-damaged tissue, rather than just the ulcer, will result in more durable reconstructive outcomes. Osteoradionecrosis of the chest wall presents as full-thickness chest wall ulcers and the involved ribs should be resected. The underlying pleura and lung may be adherent and, thus, limited lung resection may need to be performed. Reconstruction is performed with well-vascularized tissue, either local pedicled flaps or free flaps.

Negative pressure therapy utilizes subatmospheric pressure for local wound care. It provides local wound care by controlling exudate and, thus, keeping the wound clean. It is thought to promote wound healing by inducing cellular proliferation and increasing capillary blood flow. Malignancy in the wound is a contraindication to negative pressure therapy. Therefore, if suspected, malignancy should be ruled out prior to initiation of negative pressure therapy.

Hyperbaric oxygen is the administration of 100% oxygen in a pressurized chamber. This results in high tissue concentrations of oxygen, which promote neovascularization and wound healing. Hyperbaric oxygen has been shown to improve healing in soft-tissue radionecrosis and osteoradionecrosis. It can be used as an adjunct, especially when radical excision and reconstruction of radiation damaged tissue is not possible.

2020

59
Q

A 63-year-old man with a BMI of 35 kg/m2 presents with an incisional hernia. The patient underwent a midline exploratory laparotomy for trauma one year ago. Primary fascial closure was achieved with a running polypropylene suture that was performed at the time of the initial operation. CT scan shows intact rectus muscles, and the hernia defect is measured to be 10 cm at the widest, which is in the supraumbilical region. Which of the following is the most effective treatment to prevent hernia recurrence following repair?

A) Component separation with bridging mesh repair
B) Component separation with overlay mesh repair
C) Component separation with primary fascial closure
D) Component separation with retrorectus mesh repair
E) Primary fascial closure

A

The correct response is Option D.

Hernia repair is associated with a high rate of recurrence, approaching 20% in many studies. Recurrence rates are lowest when primary fascial closure of the abdominal wall is reinforced with mesh placement as an underlay.

Primary fascial closure alone or with component separation results in a higher recurrence rate than primary fascial closure with mesh reinforcement. In this example, it is unlikely that primary fascial closure would be possible, given a 10-cm hernia defect. With regard to mesh placement, there are multiple planes at which the mesh can be placed. Using a bridging repair, the mesh is used to bridge across a fascial defect and is associated with the highest rates of recurrence. In a retrorectus repair, the mesh is placed deep to the rectus (Rives-Stoppa technique) or below the transversus abdominis (transversus abdominis release technique). This is performed underneath a primary fascial closure. Conversely, in an overlay repair, the mesh is secured superficial to the abdominal wall repair. Retrorectus placement of a mesh is associated with a significantly lower recurrence rate than placement of the mesh in another position.

2020

60
Q

A 46-year-old man presents with a midline 18-cm-wide ventral hernia 1 year after undergoing midline exploratory laparotomy for a bowel resection and right end ileostomy. Medical history includes significant weight loss through diet and exercise. His weight has been stable for 2 years. BMI is 29 kg/m2. He undergoes bilateral component separation with biologic mesh bridged between the rectus muscles and concomitant panniculectomy. Which of the following clinical characteristics will most likely increase the likelihood of hernia recurrence?

A) BMI greater than 24.9 kg/m2
B) Bridged biologic mesh hernia closure
C) Concomitant panniculectomy
D) Presence of an end ileostomy
E) Prior abdominal surgery

A

The correct response is Option B.

The patient presents after significant weight loss with a wide midline ventral hernia, right end ileostomy through his rectus muscle, and an abdominal pannus. Given the 18-cm waist of the hernia defect, he is being counseled that only a bridged repair with a biologic mesh will be possible rather than total muscular coverage for the midline defect. Hernia recurrence is a major problem for patients and can be associated with specific characteristics. When the technique of bilateral component separation and inlay biologic mesh repair is being performed, the most important predictor of recurrence is whether the rectus muscle and fascia will be able to be closed at midline, creating a total submuscular repair, or whether the mesh will be bridged. A bridged repair is associated with a 33% chance of recurrence at 3 years compared to 6.2% for total muscle coverage with fascial closure at midline.

With a BMI of 29 kg/m2, the patient remains overweight despite his prior stable weight loss. Surgical site occurrences are increased in the overweight patient with a 26.4% incidence versus 14.9% in patients with BMI less than 24.9 kg/m2. Similarly, skin dehiscence is significantly increased in the overweight patient (19.3% versus 7.2%), while hernia recurrence rates are not statistically significant (11.4% versus 7.7%). Concomitant panniculectomy was associated with an increase in surgical site occurrences and skin dehiscence, but hernia recurrence rates were not affected.

Similarly, patients with existing ileostomies or stomas complicated by parastomal hernias do have a significantly increased surgical site occurrence rate (34.1% with parastomal and midline hernia versus 18.7% with midline hernia only) but hernia recurrence rates are not affected. Prior abdominal surgery will be in the clinical history of all incisional hernia patients.

2020

61
Q

A 3-year-old child with pectus excavatum deformity is evaluated for surgical correction of the chest wall. The child has experienced mild respiratory insufficiency. Which of the following is the optimal timing of treatment for this patient?

A) Surgical correction between ages 2 and 5
B) Surgical correction between ages 6 and 12
C) Surgical correction between ages 13 and 17
D) Surgical correction at skeletal maturity

A

The correct response is Option B.

Pectus excavatum is the most common congenital chest wall deformity, occurring in approximately 1 in 400 live births. The condition is more common in males, and there is a positive family history in 30 to 40% of patients. The etiology is thought to be multifactorial and associated with increased incidence of congenital cardiac abnormalities, connective tissue disorders (e.g., Marfan and Ehlers-Danlos syndromes), and scoliosis. Treatment options have shifted from the traditional open technique involving sternal osteotomy and resection of abnormal costal cartilage to minimally invasive options such as the Nuss procedure and minimally invasive technique for repair of excavatum (MIRPE), which utilizes thoracoscopy and placement of intrathoracic retrosternal support bars to reposition the sternum and allow gradual remodeling over a period of 2 to 4 years. The ideal timing of repair is mid-adolescence, usually between ages 6 and 12.

2020

62
Q

Incomplete involution of the mammary ridge during embryonic development is most likely to result in which of the following?

A) Amastia
B) Gynecomastia
C) Inverted nipple
D) Poland syndrome
E) Polymastia

A

The correct response is Option E.

The breast develops as the result of bilateral thickening of ectoderm along the milk line, or mammary ridge, from the axillary to the inguinal region. Mammary buds begin to develop as growths within the epidermis and invade the deeper mesenchyme. Much of the ridge disappears as the embryo develops as the result of apoptosis, except for the primary buds in the pectoral regions. Failure of regression of the mammary ridge can result in accessory breasts (polymastia) or accessory nipples (polythelia). Accessory breast tissue occurs in 1 to 2% of live births and commonly occurs in the axillae.

Amastia is the complete absence of the mammary gland. This occurs due to either the failure of the mammary ridge to develop or the complete involution of the mammary ridge.

Gynecomastia is defined as benign enlargement of the male breast. While pathologic cases can exist, it is most typically due to a normal response of the breast tissue to circulating levels of estrogen.

Inverted nipples are due to failure of the mesenchyme to proliferate above the level of the skin.

Poland syndrome can have the following components: hypoplasia of the breast and nipple, absence of the sternocostal portion of the pectoralis major muscle, absence of the pectoralis minor muscle, abnormalities of the chest wall, and anomalies of the upper extremity. Many etiologies have been hypothesized, with the most widely accepted being an interruption of the embryonic blood supply to the upper limb.

2020

63
Q

A 22-year-old woman presents for consideration of aesthetic breast surgery to address asymmetry. Physical examination shows a unilateral hypoplastic breast with a constricted, elevated base and a herniated nipple-areola complex. Which of the following is the most likely diagnosis?

A) Amastia
B) Micromastia
C) Poland syndrome
D) Tuberous breast
E) Virginal mammary hypertrophy

A

The correct response is Option D.

A tuberous breast is classically defined as hypoplastic with a constricted and elevated base, insufficient inferior skin, and a herniated nipple-areola complex.

Amastia would manifest without a nipple. Poland syndrome is classically described as missing the pectoralis muscle with variable breast and nipple effects. A constricted base and herniated areola are not usually associated with Poland syndrome. Hypertrophy would likely present with a broader base and increased volume. Micromastia would not manifest with a herniated areola.

2020

64
Q

A newborn is noted to have a lesion of the midline of the lower back consisting of a protruding membrane which covers meninges, cerebrospinal fluid (CSF), and neural structures. Which of the following is the primary goal of surgical repair?

A) Hydrocephalus mitigation
B) Increase in lower extremity strength
C) Infection prevention
D) Prevention of tethered cord syndrome
E) Restoration of bowel or bladder function

A

The correct response is Option C.

Meningomyelocele is the most common neural tube defect. It involves dorsal herniation of the meninges and spinal cord through the vertebrae and may produce motor and sensory nerve deficits. It is often diagnosed prenatally by elevated maternal serum alpha fetoprotein and ultrasonography. Treatment of larger defects often involves both neurosurgery and plastic surgery teams. After repair of the neural placode, the goals of soft tissue reconstruction are to cover and protect the neural element, prevent infection, and avoid any cerebrospinal fluid leak. Ideally this is performed within the first 24 to 48 hours of life. Larger defects are often best reconstructed with muscle flaps, fasciocutaneous flaps, or a combination of both. Many different flaps have been described, but considerations for adequate vascularity (such as inclusion of perforator blood vessels within geometrically designed flaps) and closure without tension are paramount.

While hydrocephalus is a common finding in patients with meningomyelocele, it is treated with cerebrospinal fluid shunting if required.

Meningomyelocele repair does not regain or improve neural abilities that are not present at birth, such as bowel and bladder function, and lower extremity motor and sensory function.

Symptoms related to tethering of the spinal cord may develop as the patient grows in as many as 20 to 50% of children who undergo meningomyelocele repair shortly after birth and many may require surgery to release the scar tissue attached to the cord. However, this condition is not prevented by meningomyelocele repair.

2020

65
Q

A 25-year-old woman who is at 16 weeks’ gestation has an elevated serum alpha-fetoprotein level. Follow-up ultrasound shows spina bifida with myelomeningocele. Which of the following is the most appropriate next step in management?

A) Gadolinium-enhanced MRI to further delineate abnormalities
B) Prenatal repair in the late third trimester
C) Repair at 1 week of age
D) Repair at 3 months of age
E) Repair within 48 hours of delivery

A

The correct response is Option E.

Postnatal repair of myelomeningocele is performed within the first 48 hours of life. Later repair is associated with worse outcomes.

Myelomeningocele is a protrusion of the meninges and spinal cord via a defect in the caudal neural tube. The higher the level of the defect, the more severe the associated abnormalities, such as bowel and bladder dysfunction, lower limb sensory and motor abnormalities, and structural abnormalities (eg, club feet). Central nervous system problems include hindbrain herniation (Chiari II malformation), tethered cord and hydrocephalus. MRI can be performed to further delineate fetal central nervous system abnormalities. Gadolinium is not given in pregnancy as it is associated with fetal morbidity and mortality.

The neurologic abnormalities caused by myelomeningocele are described by the “two-hit” hypothesis. The failure of the neural tube to close (first hit) results in exposure of the neural elements to amniotic fluid (second hit). Prenatal closure is thought to improve neurologic outcomes by decreasing the duration of exposure of neural elements to amniotic fluid. The management of myelomeningocele study (MOMS) is the only randomized-controlled trial that evaluated the outcomes of fetal surgery for myelomeningocele. The trial was stopped early because of improved outcomes in the prenatal correction group. The biggest difference was in the incidence of shunt-dependent hydrocephalus, which was 82% in the postnatal group and 40% in the prenatal group. The prenatal group also had better outcomes for mental development and motor function. However, there was a higher incidence of preterm labor and uterine dehiscence at delivery in the prenatal surgery group. Prenatal repair is performed between 19 and 27 weeks’ gestation.

Cesarean delivery is the preferred method of delivery for fetuses with myelomeningocele at many centers. This is performed when the fetus reaches term (i.e., 37 weeks’ gestation). Cesarean delivery avoids trauma to the neural tube, maintains an aseptic environment, and allows elective scheduling of meningomyelocele surgical correction.

2021

66
Q

A 54-year-old man presents for elective repair of a large, recurrent ventral hernia. Plastic surgery is consulted because of the loss of abdominal domain. Posterior component separation with placement of biologic mesh is planned. Above the level of the arcuate line, which of the following structures come together to form the posterior rectus sheath?

A) External oblique aponeurosis and internal oblique aponeurosis
B) External oblique aponeurosis, internal oblique aponeurosis, and transversus abdominis aponeurosis
C) Internal oblique aponeurosis and transversus abdominis aponeurosis
D) Transversalis fascia and peritoneum

A

The correct response is Option C.

A comprehensive understanding of the anatomy of the anterior abdominal wall is critical when performing posterior component separation. Above the arcuate line, the anterior rectus sheath is derived from the external oblique aponeurosis and the anterior component of the internal oblique aponeurosis. The posterior rectus sheath is derived from the posterior component of the internal oblique aponeurosis and the aponeurosis of the transversus abdominis muscle. The rectus abdominis muscle sits between the anterior and posterior rectus sheaths superior to the arcuate line. Below the arcuate line, the anterior rectus sheath is derived from the aponeuroses of all three muscles: the external oblique, the internal oblique, and the transversus abdominis muscles. The posterior rectus sheath does not exist below the arcuate line. The rectus muscle is only separated from the abdominal viscera by the transversalis fascia and the peritoneum.

2021

67
Q

A 67-year-old woman with a large ventral hernia after prior exploratory laparotomy for trauma is scheduled for complex abdominal hernia repair with posterior components separation and mesh. The planned surgical technique involves division of which of the following muscles and locations?

A) External abdominal oblique lateral to the linea semilunaris
B) External abdominal oblique medial to the linea semilunaris
C) Internal abdominal oblique lateral to the linea semilunaris
D) Transversus abdominis lateral to the linea semilunaris
E) Transversus abdominis medial to the linea semilunaris

A

The correct response is Option E.

The posterior components separation technique involves dissection in the retro-rectus plane to release the transversus abdominis muscle medial to the linea semilunaris, allowing medialization of the posterior rectus sheath-transversalis fascia complex. This fascial layer is approximated to form the posterior layer of closure. The anterior layer, consisting of abdominal wall musculature, is approximated at the linea alba. Mesh is usually placed in between these two layers.

In the more commonly performed anterior components separation technique, the external abdominal oblique is released lateral to the linea semilunaris to permit medialization of the rectus abdominis muscle for abdominal closure. The internal abdominal oblique is not released in anterior component separation. The internal abdominal oblique fascia is released medial to the semilunaris in posterior component separation.

2021

68
Q

A 28-year-old man presents for consultation about the excess tissue of his chest. Medical history includes no weight change and no comorbidities. BMI is 26 kg/m2. Physical examination shows the nipple position is preserved in the craniocaudal dimension and firm soft tissue underlying the central breast bilaterally. Which of the following is the most appropriate surgical approach?

A) Horizontal scar breast reduction with nipple-areola grafting
B) Inverted-T pattern breast reduction with inferior pedicle
C) Liposuction alone
D) Periareolar access for open central glandular excision with liposuction
E) Skin-sparing mastectomy with nipple-areola tattoo

A

The correct response is Option D.

Scar minimization is a priority when possible in patients with gynecomastia. The patient’s acceptable BMI, absence of weight change, and physical description imply minimal skin excess, making him a candidate for limited scar techniques. Direct excision of the central gland through periareolar incision with liposuction for marginal contouring would address the patient’s condition without excess scarring. Liposuction alone is unlikely to adequately address the firm central glandular tissue under the nipple-areola complex. The other techniques such as inverted-T pattern breast reduction with inferior pedicle, horizontal scar breast reduction with nipple-areola grafting, and skin-sparing mastectomy with nipple-areola tattoo would all unnecessarily increase scarring.

2021

69
Q

An 18-year-old woman presents for surgical repair of an under-developed left breast. On physical examination, the left side shows concave chest wall, absent anterior axillary fold, and a hypoplastic and superiorly displaced nipple. The left upper extremity is most likely to demonstrate which of the following findings in this patient?

A) Acrosyndactyly
B) Arachnodactyly
C) Macrodactyly
D) Polydactyly
E) Symbrachydactyly

A

The correct response is Option E.

The patient described meets criteria for Poland syndrome. This complex includes breast and/or nipple-areola complex hypoplasia, absence of the sternal head of the pectoralis major, rib abnormalities, and syndactyly or symbrachydactyly, fused and shortened digits. Polydactyly (multiplication of digits), macrodactyly (overgrowth and enlarged digits), arachnodactyly (long fingers), and acrosyndactyly (fusion of distal aspects of digits only) are not usually associated with Poland syndrome.

2021

70
Q

An 18-year-old woman with right-sided Poland syndrome requests improvement in the appearance of her chest. Physical examination shows absence of an anterior axillary fold on the right side and a pectus excavatum deformity with an overlying hypoplastic right breast. The nipple-areola complex is small, lateral, and raised by about 3 cm compared with the left side. Which of the following thorax, breast, nipple-areola complex (TBN) classifications best characterizes this patient’s Poland syndrome deformity?

A) T1, B1, N1
B) T1, B2, N1
C) T2, B1, N2
D) T3, B2, N3

A

The correct response is Option C.

Poland syndrome is likely a multifactorial genetic syndrome related to the embryologic timing and development of the subclavian arch. Its unifying finding is absence of the pectoralis major muscle but is variable in other manifestations of underdevelopment of the chest wall, breast, and ipsilateral upper extremity. Poland syndrome is most often reported as more frequent in males, but some series show equal expression in males and females. Additionally, up to 10% of patients may have associated dextrocardia.

Having a way to describe or classify a deformation in an organized fashion is helpful in planning reconstruction, determining results and outcomes, and discussing cases with colleagues. The thorax, breast, nipple-areola complex (TBN) system was proposed and published by Romanini et al. to do just that. Since that publication, further research by the group based on the TBN classification has been published and others have suggested modifications to include the presence or absence of ipsilateral upper extremity anomalies.

Thorax
T1: absence of all or part of pectoralis
T2: T1 + pectus excavatum or carinatum
T3: T1 + rib aplasia (usually 3 and 4)
T4: T1 + T2 + T3

Breast
B1: hypoplastic breast
B2: breast aplasia (amastia)

Nipple-areola complex
N1: hypoplastic NAC less than 2 cm displaced
N2: hypoplastic NAC more than 2 cm displaced
N3: athelia

The patient in the scenario is best described by the TNB classification as T2 (no pectoralis, pectus excavatum), B1 (breast hypoplasia) and N2 (hypoplastic NAC greater than 2 cm displaced). According to this classification, she is probably best served by correction of the pectus first, then correction of the breast.

2021

71
Q

A 55-year-old man presents with ventral incisional hernia of the abdomen. BMI is 32 kg/m2. Medical history includes an exploratory laparotomy following a traumatic injury 1 year ago. The abdominal fascia was closed primarily at the end of the procedure. Physical examination shows a fascial deficit 10 cm in width. CT scan demonstrates that rectus muscles are intact bilaterally and 12 cm apart in the periumbilical region. Hernia repair is performed, but the fascia cannot be brought together primarily at the midline. Which of the following operations is most likely to result in the lowest risk for future hernia formation in this patient?

A) Bilateral component separation
B) Bilateral component separation with onlay mesh reconstruction
C) Bilateral component separation with underlay mesh reconstruction
D) Inlay mesh reconstruction
E) Unilateral component separation

A

The correct response is Option C.

Long-term hernia risk is lowest following primary fascial closure and placement of mesh, either in the retrorectus position or as an underlay. Inlay mesh reconstruction, in which primary fascial closure is not possible and the mesh serves as a bridge, is associated with the highest rates of abdominal hernia formation. Unilateral or bilateral component separation may allow for primary fascial closure, but placement of a mesh augments the repair and reduces hernia recurrence. Regarding onlay mesh placement, in which the mesh is placed superficial to the fascia, hernia recurrence rates have been shown to be higher in obese patients than when the mesh is placed as an underlay.

2022

72
Q

A 50-year-old woman with sarcoma of the right lateral chest wall has completed neoadjuvant radiation. Surgical resection is performed and results in a 6-cm diameter skeletal defect of the chest wall. Soft-tissue reconstruction only is planned. Soft-tissue only reconstruction is indicated because of which of the following factors?

A) Lateral defects do not need skeletal reconstruction
B) Radiation is a contraindication for skeletal chest wall reconstruction
C) Radiation renders the chest wall stiff, and thus large skeletal defects are well tolerated
D) Reconstruction should be staged after margin confirmation on final pathology
E) Skeletal reconstruction should not be performed in oncologic resections

A

The correct response is Option C.

The goals of skeletal chest wall reconstruction are restoration of chest wall stability and protection of intrathoracic structures. Large chest wall defects can result in flail segments with impairment of ventilatory mechanics. The lateral thoracic wall is most susceptible to such alterations since it the most mobile part of the chest wall. There is general consensus that skeletal chest wall reconstruction is indicated if four or more ribs are resected or if lateral defects are ? 5 cm. Total sternectomy defects disrupt the chest wall ring and increase dependence on abdominal breathing and may cause chronic chest wall pain; as such, they are considered by some to be indications for skeletal reconstruction. Posterior chest wall defects under the scapula (above the fourth rib) and anterior chest wall wounds under the pectoralis major do not require skeletal reconstruction since sufficient rigidity and protection is provided by these structures. Posterior defects around the area of the scapular tip may cause scapular entrapment and thus require skeletal reconstruction. Radiation fibrosis renders the chest wall stiff, and thus larger skeletal defects are well tolerated. Oncologic resections are not a contraindication for skeletal reconstruction. Skeletal reconstruction is most commonly performed with prosthetic materials, like synthetic meshes, biologic meshes and osteosynthesis materials. Radiation is not a contraindication for skeletal chest wall reconstruction. Reconstruction is performed at the same time as resection.

2022

73
Q

A 31-year-old man who recently had a 100-lb (45.4-kg) weight loss presents with a 5-year history of bilateral gynecomastia. Results of endocrine workup are negative, and testicular ultrasonography shows no abnormalities. He does not take any hormones. Physical examination shows an estimated 300 g of primarily glandular tissue and marked skin redundancy. Which of the following is the best treatment for the degree of gynecomastia in this patient?

A) Direct gland and skin excision with liposuction
B) Direct gland and skin excision with nipple transposition
C) Direct gland excision and cryolipolysis
D) Liposuction and direct gland excision only
E) Liposuction and nipple transposition

A

The correct response is Option B.

This patient has class IIAii gynecomastia per the McMaster classification of gynecomastia. To treat this degree of gynecomastia, in which there is 250 to 500 g of tissue that is primarily glandular and associated with marked skin redundancy, direct gland and skin excision with nipple transposition (with or without liposuction) is indicated. Skin excision patterns include a boomerang pattern to correct vertical and horizontal skin excess primarily in patients with massive weight loss. Transverse incisions are used to remove vertical excess.

Liposuction would not address the glandular tissue, and skin excision is needed to treat the skin redundancy. Liposuction combined with direct gland excision also does not correct the skin redundancy. Cryolipolysis is indicated for fat reduction, which would not treat the primarily glandular component. Direct gland and skin excision with liposuction does not correct the nipple position.

2022

74
Q

A 33-year-old woman seeks panniculectomy to address intertrigo following a 100-lb weight loss after undergoing bariatric surgery 18 months ago. Her weight has been stable for the past 3 months, and her current BMI is 30 kg/m2. Which of the following aspects of this patient’s history is most likely to interfere with insurance coverage?

A) BMI of 30 kg/m2
B) 3 Months of weight stability
C) 18 Months status postbariatric surgery
D) Primary symptom of intertrigo
E) 100-lb weight loss

A

The correct response is Option B.

Many patient history factors are important when considering indications for body contouring after weight loss. Surgical indications include symptomatic rashes, large amounts of weight loss, adequate time between bariatric surgery and body contouring surgery, decreased BMI, and a substantial time period of weight stability, longer than 3 months. Additionally, insurance carriers have varying criteria to allow authorization of abdominal contouring procedures, specifically panniculectomy. Many insurance carriers require 6 months of weight stability. Severe intertrigo, 100-lb weight loss, 18 months status post bariatric surgery, and a relatively low BMI would be in keeping with frequently used clinical indications for surgery and insurance coverage criteria.

2022

75
Q

A 55-year-old obese woman presents with sternal wound dehiscence 10 days after undergoing pectoralis major flap reconstruction. Medical history includes stable hypertension, coronary artery disease, and coronary artery bypass grafting complicated by mediastinitis. Which of the following factors is a predictor for recurrent sternal wound dehiscence in this patient?

A) Age
B) Coronary artery disease
C) Female sex
D) Hypertension
E) Pectoralis major flap harvest

A

The correct response is Option C.

A retrospective chart review of 77 patients was conducted over a 7-year period, focusing on patients with sternum dehiscence who underwent pectoralis major transposition for sternal reconstruction. Female sex, smoking, detachment of the humeral insertion, and operation time are associated with postoperative wound complications.

Breast size in women was previously found to be a risk factor for the development of post-sternotomy mediastinitis. Jones et al. found that recurrent dehiscence after tissue-flap coverage of the sternum occurs more frequently in obese women with large pendulous breasts. An association between female sex and recurrent wound dehiscence was also found in the study. This might have been caused by traction of large breasts on the wound edges or devascularization of mammary tissue. Due to the retrospective nature of this study, breast size as a cofactor could not be included in the analysis.

Age, pectoralis major harvest, hypertension, and coronary artery disease are not independent risk factors for recurrent sternotomy dehiscence.

2022

76
Q

Absence of which of the following structures is most characteristic of patients with Poland syndrome?

A) Clavicular head of the pectoralis major muscle
B) Ipsilateral ribs
C) Pectoralis minor muscle
D) Serratus anterior muscle
E) Sternocostal head of the pectoralis major muscle

A

The correct response is Option E.

What follows is the description from the original records found in Paris, France: “In 1841, Alfred Poland, a 19-year-old medical student at Guy’s Hospital in London, England, published the classic description of the syndrome that bears his name. Serving as an anatomy demonstrator, Poland recorded that, in his subject: ‘the whole of the sternal and costal portions of the pectoralis-major muscle were deficient; but its clavicular origin quite normal. In the left hand, the middle phalanges were absent. The web between the fingers extended to the first articulation the hand was shorter than the right the left thumb was quite normal.’ In a footnote, he indicated that ‘the hand has been deposited in the Museum of Guy’s Hospital.’ Poland was a popular teacher and later pursued a distinguished surgical career.” (Charlier P, Deo S, Galassi FM, Benmoussa N. Poland syndrome before Alfred Poland: the oldest medical description [Paris, France, 1803]. Surg Radiol Anat. 2019;41[10]: 1117-1118.)

Today, Poland syndrome presents a spectrum of chest wall anomalies ranging from simple to complex. These deficiencies are largely cosmetic, with the most common (simple) form presenting as a unilateral absence of the sternocostal head of the pectoralis major muscle. The deformity can also be complex, with ipsilateral absence of ribs, axillary webbing, and foreshortening of the hemithorax. Reconstructive options for the chest wall depend on anatomical severity, gender, associated anomalies, and, of course, the patient’s preference.

Clavicular head of pectoralis major muscle is generally present in these patients. Pectoralis minor, absence of serratus anterior, and rib absence may or may not be present in patients with Poland syndrome.

2022

77
Q

A 35-year-old transgender man (assigned female at birth) comes to the office to discuss top surgery for a masculine chest appearance. On examination, the patient has D-cup breasts and Grade III ptosis. Which of the following is the most appropriate surgical option for this patient?

A) Circumvertical reduction mammaplasty
B) Liposuction only
C) Mastectomy with free nipple graft
D) Nipple-sparing mastectomy
E) Wise-pattern reduction mammaplasty

A

The correct response is Option C.

The most appropriate surgical procedure in this patient is a subcutaneous mastectomy with free nipple graft due to breast size and ptosis. Top surgery, or excision of the female breast tissue and shaping of the male chest, is often the first surgical procedure for female-to-male transgender patients. This marks the beginning of the patient’s surgical transition into a masculine phenotype and is associated with profound and impactful psychological and aesthetic benefits for the individual. The goal of top surgery is to produce a normal-appearing male chest. This is achieved by removing the breast parenchyma, obliterating the inframammary fold, decreasing the areolar size, and moving the nipple-areola complex into the appropriate position.

There are numerous studies demonstrating excellent results and high patient satisfaction with subcutaneous mastectomy and free nipple grafts in patients with large and ptotic breasts. Additionally, patients with a BMI over 27 kg/m2and a distance from nipple to inframammary fold greater than 7 cm demonstrate better outcomes with fewer complications. Nipple-sparing mastectomy is inappropriate due to the degree of breast ptosis and the amount of skin that needs to be removed.

Liposuction alone will not achieve the goals above and will result in residual breast tissue and excess skin. Additionally, liposuction will not address areolar size or position. Liposuction is useful when combined with other techniques to help feather tissue thickness and contour the edges of the chest. Both Wise-pattern and circumvertical reduction mammaplasties will leave behind too much breast tissue and will not create the desired masculine chest appearance.

2023

78
Q

A 48-year-old transgender woman (assigned male at birth) is evaluated by an endocrinologist for hormone therapy to induce breast development. History is significant for factor V Leiden mutation. Which of the following hormone therapies would put this patient at greatest risk for developing deep venous thrombosis?

A) Oral ethinyl estradiol
B) Oral progesterone
C) Oral tamoxifen
D) Subcutaneous testosterone
E) Transdermal estradiol

A

The correct response is Option A.

An aim of estrogen therapy in transgender women is to induce feminization through breast formation. Estrogen is responsible for breast tissue proliferation, whereas progesterone causes differentiation in female breast development. No evidence exists that the addition of progesterone improves breast development among male-to-female (MtF) patients.

Estrogen is a prothrombotic hormone. The patient’s underlying venous thromboembolic risk profile should be evaluated prior to initiating estrogen therapy. Both oral estradiol and ethinyl estradiol exert an increased prothrombotic risk due to hepatic first-pass metabolism. Compared with oral estradiol, the unique molecular structure of ethinyl estradiol is associated with greater prevalence of venous thromboembolism (VTE) in MtF patients. Transdermal estradiol is associated with the lowest VTE risk in transgender women.

Testosterone therapy induces virilization in female-to-male transgender persons, including development of male physical contours, reduction of breast glandular tissue, and increase in muscle mass and body fat redistribution. Subcutaneous administration of testosterone is the preferred route of treatment by transgender men as compared with intramuscular administration. Testosterone therapy is not associated with venous thromboembolism, but long-term testosterone undecanoate has a potential risk for oil pulmonary embolism. Venous thromboembolism is a complication generally only observed in male-to-female transgender persons on estrogen therapy.

Oral tamoxifen is not a therapy used to induce breast development. It is used for breast cancer risk reduction

2023

79
Q

A 44-year-old woman comes to the clinic because of chronic intermittent infection, pain, and drainage of the right axilla. History is significant for obesity, type 2 diabetes, and smoking, as well as multiple courses of antibiotic therapy and abscess drainage. A photograph is shown. Which of the following skin appendages is most likely involved in the disease process in the photograph?

A) Apocrine glands
B) Eccrine glands
C) Hair follicles
D) Sebaceous glands
E) Stratum spinosum

A

The correct response is Option A.

This patient has hidradenitis suppurativa, which is an inflammatory disease that arises in the apocrine glands. Apocrine sweat glands are primarily located in the axilla, perineum, groin, and inframammary region. These glands produce a viscid, lipid-rich, milky fluid that functions primarily as a pheromone and produces body odor. Occlusion of the glands results in inflammation and subcutaneous abscess formation with resultant odor, pain, and drainage. Early disease is typically treated with antibiotic therapy, local and systemic steroids, antiandrogen therapy, and botulinum toxin injections. More severe cases may benefit from tumor necrosis factor alpha inhibitors, such as infliximab or etanercept. Surgical treatment is common and may range from simple drainage of the abscess to wide excision with skin graft or flap reconstruction.

Eccrine glands are found throughout the body and are responsible for thermoregulation by secretion of a thin, clear, hypotonic fluid during sweating. There is some recent data that suggests hair follicle occlusion and folliculitis may lead to apocrine occlusion; however, hair follicles are found throughout the body without evidence of clinical hidradenitis in those locations. Sebaceous glands are found throughout the skin except for the palms and soles and produce sebum, which is an oily substance that lubricates skin and hair follicles. These glands are particularly abundant in the head and neck region. The stratum spinosum is a layer of the epidermis located between the stratum granulosum and stratum basale.

2023

80
Q

A 28-year-old woman is evaluated to discuss treatment options for a chronic history of bilateral axillary purulent drainage from multiple sinus tracts. History includes acne. She has smoked 1 pack of cigarettes daily for 10 years. BMI is 25.1 kg/m2. Which of the following factors in this patient is most commonly associated with increased severity of symptoms of her condition?

A) Age
B) BMI
C) Duration of disease
D) Sex
E) Smoking

A

The correct response is Option E.

Tobacco smoking is known to have negative effects on the integument, including premature skin aging, abnormal wound healing, and exacerbation of chronic dermatoses, such as hidradenitis suppurativa (HS), psoriasis, and cutaneous lupus erythematosus.

HS is a debilitating chronic inflammatory disease of the folliculopilosebaceous units of the apocrine gland-bearing areas of the skin. The disease is characterized by physical discomfort and malodorous drainage from nodules, abscesses, scars, and sinus tracts in the axillae, groin, perineum, buttocks, and inframammary areas. HS presents most commonly in the third and fourth decades of life, with greater prevalence among women (3:1 female-to-male ratio) and patients of African descent. The severity of HS is not linked to the sex of the patient or duration of HS disease. Risk factors for HS are obesity, autoimmune diseases, and smoking.

A significant association has been established between smoking and HS. Smoking has also been associated with a dose-response relationship to clinical HS disease severity. The pathophysiologic mechanism of smoking on HS arises from modification in the skin microflora, and nicotine-related increased activity of sweat glands and hyperplasia of epidermal and follicular occlusion.

2023

81
Q

An 18-year-old transgender man (assigned female at birth) presents for an initial consultation to plastic surgery to discuss top surgery. He has been taking testosterone shots for the past year. The patient wants to know the preoperative requirements for gender affirmation surgery. Which of the following best describes the 7th Edition World Professional Association for Transgender Health (WPATH) criteria for top surgery?

A) The patient must be at least 16 years of age to provide informed consent
B) The patient must be on hormone therapy for one year prior to surgery
C) The patient must have persistent, well-documented gender dysphoria
D) The patient must not have any other mental health concerns

A

The correct response is Option C.

The 7th Edition World Professional Association for Transgender Health (WPATH) criteria for top surgery is that the patient must have persistent, well-documented gender dysphoria. The WPATH guidelines for female-to-male top surgery also require that the patient has the capacity to make a fully informed decision and consent for treatment and is the age of majority in a given country. Additionally, if significant medical or mental health concerns are present, they must be reasonably well-controlled.

Hormone therapy is not a prerequisite for female-to-male top surgery. Though many patients do take hormone therapy before surgery, it is not required to undergo surgery. The age of majority varies from country to country to provide informed consent. In the United States, 18 years of age is considered the age of majority. Many patients with gender dysphoria have other mental health concerns. As long as comorbid conditions are well-controlled, such patients are still acceptable candidates for surgery.

2023

82
Q

A 38-year-old man presents with a large midline abdominal ventral hernia following colectomy. Physical examination shows a 16 × 9-cm midline fascial defect. Posterior component separation repair with transversus abdominis release and retrorectus mesh placement is planned. To maintain innervation to the rectus abdominis muscles, identification and preservation of which of the following nerves during dissection is most appropriate?

A) Iliohypogastric
B) Ilioinguinal
C) Intercostal
D) Lateral cutaneous
E) Subcostal

A

The correct response is Option C.

Component separation repair of abdominal wall hernias can be performed via both the classic anterior component separation technique and the posterior component separation technique. The posterior component separation repair is based on the Rives-Stoppa retrorectus technique. This technique uses mesh placement in the 6- to 8-cm–wide space between the posterior rectus sheath and the rectus muscle. This plane for mesh placement has been found to have the lowest risk for hernia recurrence.

Extending the dissection lateral to the lateral border of the posterior rectus sheath via a transversus abdominis release allows for tension-free repair of larger defects. Hernia recurrence rates as low as 3 to 6% have been documented with this technique.

A key maneuver during this dissection is the identification of the intercostal nerves, which innervate the rectus abdominis muscles. They should be identified and preserved as they enter the posterior aspect of the rectus muscle just medial to the linea semilunaris.

The lateral cutaneous nerves innervate the skin overlying the bilateral hypogastric regions.

The ilioinguinal nerve extends from the inferior branch of the anterior ramus of L1. It travels through the inguinal canal. If this nerve is damaged during inguinal hernia repairs, it could lead to loss of sensation in the inguinal region, mons pubis, medial thigh, and the anterior scrotum or labia majora.

The iliohypogastric nerve is the continuation of the superior branch of the anterior ramus of L1. It innervates the transversus abdominis and internal oblique muscles. It is also responsible for the sensation of the skin overlying the iliac crest, superior inguinal canal, and hypogastric regions. The subcostal nerve travels along the border of the twelfth rib until it passes into the abdominal wall and runs through the second and third muscular layers.

The subcostal nerve innervates the inferior portion of the external oblique muscle, skin, region superior to the inguinal crest, and the region inferior to the umbilicus. The subcostal nerve does not innervate the rectus abdominis muscle.

2023

83
Q

An otherwise healthy 13-year-old boy is brought to the plastic surgeon’s office because of a 6-month history of bilateral breast enlargement. The patient takes no medications. There is no family history of breast cancer. Examination shows bilateral, 3-cm, firm, rubbery, subareolar masses without nipple discharge. No testicular masses are noted. Workup shows no other abnormalities. The patient reports that he is afraid of being teased. Which of the following is the most appropriate management?

A) Direct surgical excision
B) Initiation of tamoxifen
C) Liposuction
D) Mammography
E) Observation

A

The correct response is Option E.

Adolescent gynecomastia is common, and there is often spontaneous regression after 6 months. While the cause is often unknown, hormonal influences are thought to play a significant role. Because the overwhelming majority of pubertal gynecomastia regresses over the course of several years, observation is the mainstay of treatment.

Mammography is not necessary if the history and physical examination findings are consistent with physiologic gynecomastia, and it is often unhelpful in adolescent gynecomastia. While tamoxifen can be effective for gynecomastia and some studies have shown safe use in adolescent boys, it is not a first-line treatment for mild disease as described in this case. It has been used for severe disease, but it is not FDA-approved for this purpose.

Surgical treatment, whether liposuction or surgical excision, should be reserved for failure of conservative management.

2023

84
Q

A 20-year-old woman presents with a right-sided sunken anterior chest wall, hypoplasia of the breast, superiorly placed nipple-areola complex, normal pectoralis muscle, and normal sternal position. Which of the following is the most likely diagnosis?

A) Anterior thoracic hypoplasia
B) Breast hypoplasia
C) Poland syndrome
D) Scalp-ear-nipple syndrome
E) Tuberous breast deformity

A

The correct response is Option A.

The patient has anterior thoracic hypoplasia. It is a rare disorder and separate from Poland syndrome. It presents with posterior displacement of the ribs, an anteriorly unilateral sunken chest wall, hypoplasia of the ipsilateral breast, and a superiorly displaced nipple-areola complex. However, the sternum and pectoralis major are normal.

Poland syndrome is characterized by various degrees of thoracic and ipsilateral upper extremity anomalies, including some or all of the following: absence or hypoplasia of the breast, absence of the nipple, absence of the pectoralis major or minor, absence of adjacent muscles and sometimes costal cartilage, rib abnormalities, and upper extremity deformities (eg, syndactyly, brachydactyly, or micromelia).

Tuberous breast deformity is a spectrum of aberrant breast shape, characterized by a high and tight inframammary fold, enlarged diameter of the nipple-areola complex, herniation of breast parenchyma through the nipple-areola complex, and most often breast asymmetry.

Scalp-ear-nipple syndrome is characterized by a triad: areas of hairless raw skin over the scalp (present at birth and healing during childhood), prominent hypoplastic ears with nearly absent pinnae, and bilateral amastia.

Breast hypoplasia refers to underdeveloped breasts.

2023

85
Q

A pregnant woman in the second trimester undergoes ultrasonography for anatomic screening. The screening discloses that the fetus has an omphalocele without liver involvement. Which of the following chromosomal findings is most likely in this fetus?

A) 5p monosomy
B) 22q11 microdeletion
C) Trisomy 13
D) 47,XXY

A

The correct response is Option C.

An omphalocele is a congenital abdominal wall defect through the umbilical ring, which contains varying amounts of abdominal viscera and organs covered by a fused membrane of infant peritoneum, Wharton jelly, and amnion into which the umbilical cord inserts. As indicated, giant omphaloceles contain more than intestines and have a large diameter that laterally displaces the rectus muscles as they insert onto the ribs. Once a giant omphalocele is diagnosed, antenatal counseling about the natural history of these malformations is provided. Considerations for the best course for the fetus, pregnancy, delivery, and neonatal period can be determined. Part of that counseling includes the high rate of aneuploidy of autosomes (30%) such as in trisomy 13 in these patients. Pediatric plastic surgeons may be asked to be involved with larger lesions to help reconstruct the abdominal wall. Timing and needs will be determined by pulmonary capacity and maturity, the presence of other anomalies, loss of domain, and reconstructive needs. The plastic surgeon should be conversant with all aspects prior to joining in antenatal or neonatal counseling.

22q11 microdeletion has many syndromic names, including velocardiofacial syndrome or DiGeorge syndrome, and is related to cleft palate.

47,XXY is Klinefelter syndrome and is not associated with omphalocele. Plastic surgeons may see these patients in adolescence with reports of gynecomastia.

5p monosomy is cri du chat syndrome and is also not associated with omphalocele. Plastic surgeons may be asked to evaluate facial differences, including small jaw and hypertelorism.

2023

86
Q

A male newborn with a large lumbar bulge receives a diagnosis of myelomeningocele. Which of the following findings supports the diagnosis of myelomeningocele over meningocele?

A) Absence of neural structure involvement
B) Alpha-fetoprotein concentration within the normal range
C) Minimal neurological symptoms
D) Presence of hydrocephalus

A

The correct response is Option D.

Meningocele and myelomeningocele are neural tube defects resulting in spinal dysraphism. Both are forms of spina bifida, with myelomeningocele being more severe due to the protrusion and underdevelopment of neural structures and meningocele only involving the meninges, commonly referred to as spina bifida occulta. Cerebellar tonsillar herniation with attendant hydrocephalus is part of myelomeningocele only. Because only the dural sac is involved in meningocele, neural deficits are milder. Alpha-fetoprotein concentration is tested in the first trimester of pregnancy.

2023